Top Banner
THEME : Chest pain A Oral analgesia B Oral analgesia and admission C Chest drain and analgesia Rib fractures may be extremely painful. The importance of treating rib fractures is in the management of the underlying lung and the prevention of pulmonary complications such as pneumonia and collapse. Atelectasis will occur if the lung is not aerated fully, which would occur if there is poor chest expansion due to pain. Young patients can tolerate a rib fracture more easily than elderly patients. With good physiotherapy and analgesia, these young patients may be managed conservatively at home. However, elderly patients would not tolerate rib fractures so well and necessitate admission with regular analgesia, physiotherapy and probably prophylactic antibiotics. In a polytrauma patient, it would be wise to insert a chest drain for both the treatment of the pneumothorax and to prevent the development of a recurrent or tension pneumothorax when the patient is ventilated. Scenario 1 Young footballer with fracture of the fourth rib, normal chest X-ray. A Correct answer Scenario 2 Elderly man with fracture of the fourth and normal chest X-ray. A B Correct answer
110
Welcome message from author
This document is posted to help you gain knowledge. Please leave a comment to let me know what you think about it! Share it to your friends and learn new things together.
Transcript
Page 1: 51 100

THEME: Chest painA Oral analgesiaB Oral analgesia and admissionC Chest drain and analgesia

Rib fractures may be extremely painful. The importance of treating rib fractures is in the management of the underlying lung and the prevention of pulmonary complications such as pneumonia and collapse. Atelectasis will occur if the lung is not aerated fully, which would occur if there is poor chest expansion due to pain. Young patients can tolerate a rib fracture more easily than elderly patients. With good physiotherapy and analgesia, these young patients may be managed conservatively at home. However, elderly patients would not tolerate rib fractures so well and necessitate admission with regular analgesia, physiotherapy and probably prophylactic antibiotics. In a polytrauma patient, it would be wise to insert a chest drain for both the treatment of the pneumothorax and to prevent the development of a recurrent or tension pneumothorax when the patient is ventilated.

Scenario 1

Young footballer with fracture of the fourth rib, normal chest X-ray.

A Correct answer

Scenario 2

Elderly man with fracture of the fourth and normal chest X-ray.

A

B Correct answer

Scenario 3

Polytrauma – fracture of the fourth rib and femur, and pneumothorax.

A

C Correct answer

Page 2: 51 100

THEME: Investigation and management of gastrointestinal perforation A Cervical exploration and drainageB Colonic resection and primary anastomosisC Conservative treatment (antibiotics, total parenteral nutrition,          nasogastric tube)D Computed tomography scan +/- guided drainageE Emergency gastrectomyF Emergency thoracotomyG Gastrograffin enemaH Gastrograffin swallowI Hartmann’s procedureJ Oesophageal resectionK Oesophageal stentingL Omental patch repairM Primary repair and drainage (oesophagus)N Upper gastrointestinal endoscopy 

The above are all methods of evaluating or managing gastrointestinal perforation. For each of the following clinical presentations please choose the most appropriate answer from the list. Each item may be used once, more than once, or not at all.

 

Scenario 1

A previously fit 50-year-old man presents to The Emergency Department complaining of retrosternal pain radiating to his back, on a background of 24 h of dysphagia. He tells you that he underwent an upper gastrointestinal endoscopy for chronic dyspeptic symptoms 3 days earlier. Apparently the procedure had been difficult, requiring multiple attempts to intubate the oesophagus, before being abandoned. On examination he has mild crepitus in the soft tissues around his neck and auscultation reveals a positive Hamman’s sign. His temperature is 38.3°C, blood pressure 130/75 mmHg, pulse 105/min. Water-soluble contrast swallow shows a small localised leak in the region of the thoracic oesophagus.

A

C Correct answer

C – Conservative treatment (antibiotics, total parenteral nutrition, nasogastric tube)

Page 3: 51 100

There remains significant contention about operative versus non-operative management for oesophageal perforation. This is principally because surgical intervention implies the need for thoracotomy, a major plus procedure with high morbidity and lengthy recovery. There is general agreement that in the conservative management of instrumental oesophageal perforations, selection criteria should include small, localised leaks, with ready drainage back into the oesophagus (following contrast radiography), no pleural contamination and low-grade symptoms, with an absence of systemic signs of sepsis. As this man has presented late (> 3 days), he has theoretically undergone a selection process of sorts (ie if his perforation was more severe he would have deteriorated more rapidly). Conservative management includes the aggressive use of broadspectrum antibiotics, parenteral nutrition and nasogastric suction with gastric acid suppression. Undoubtedly any clinical deterioration may cause revision of the plan for conservative therapy.

Scenario 2

A 64-year-old woman presents to Casualty with a 24-h history of progressive left iliac fossa pain. She has been conservatively treated for diverticulitis in the past (with prior barium enema confirming sigmoid disease). On examination, she has localised tenderness and guarding in the left iliac fossa. In addition, her temperature is 38.9°C, blood pressure 105/60 mmHg, pulse 110/min.

A

D Correct answer

D – Computed tomography (CT) scan abdomen +/- guided drainage

This woman is demonstrating signs and symptoms of septically complicated diverticular disease (arising from micro- or macroperforation). The management depends on the clinical scenario – simple diverticulitis does not usually require urgent radiological assessment and can be managed with bowel rest and antibiotics. The more ill patient, as in this case, should undergo contrast CT scanning to exclude free perforation and facilitate drainage of any abscess. The patient with generalised peritonitis in general requires a laparotomy.

THEME: Congenital neck lumpsA LaryngocoeleB Thyroglossal cystC Branchial cystD Desmoid cystE Dermoid cystF LymphangiomaG Chemodectoma

Page 4: 51 100

For each of the patients below, chose the most appropriate diagnosis from those above. Each may be used once, more than once or not at all.

 

Scenario 1

A 55-year-old gentleman presented with a long-standing history of mild stridor and hoarseness that had suddenly worsened. On palpation a large soft swelling over the thyrohyoid membrane was felt, when pressure was applied, this swelling disappeared.

A Correct answer

Laryngocoele

When the laryngeal saccule is expanded with air it is termed a laryngocoele. These may spread superiorly and present in the false cord (internal laryngocoele) or may pass through the thyrohyoid membrane and present as a lump in the neck.

Scenario 2

A 7-year-old girl presented with a painless cystic swelling anterior to the thyroid cartilage. The swelling was transilluminable, mobile in all directions and moved on protrusion of the tongue.

A

B Correct answer

Thyroglossal cyst

A thyroglossal cyst is a fluctuant midline swelling which moves on tongue protrusion. It develops in cell nests in the thyroid gland’s migration path.

THEME: Lumps in the neck A Thyroglossal cystB Solitary thyroid noduleC Thyroid nodule in multinodular goitreD Cervical nodal metastasisE Branchial cystF Cystic hygromaG Deep neck space abscess

Page 5: 51 100

H Salivary gland swellingI Cervical lymphadenopathy 

From the scenarios below, choose the most likely diagnosis from the list above. Each diagnosis may be used once, more than once or not at all.

Patient demographics are very important when making a diagnosis. Hence the 18-year-old smoker is likely to have an infection, whereas a 54-year-old is more likely to have a carcinoma. The hard mass in the floor of the mouth represents a stone in the submandibular gland. Patients with a goitre are more likely to develop compression symptoms; solitary thyroid nodules are usually small and non-functioning. Cystic hygromas are rare, congenital abnormalities of lymphoid tissue that may expand rapidly causing life-threatening airway compromise. Branchial cysts may enlarge periodically due to infection, but are not associated with airway difficulty. Tonsillar carcinoma is a common cause of cervical nodal metastasis, and unilateral enlargement always raises suspicion in smokers.

Scenario 1

A 33-year-old non-smoker who presents with a 6-month history of recurrent, intermittent left-sided neck swelling associated with pain and dry mouth. He can feel a hard mass in the floor of his mouth.

A

H Correct answer

Scenario 2

A 25-year-old female who presents with mild dysphagia and a palpable mass on the left of the midline.

A

C Correct answer

Scenario 3

Page 6: 51 100

An 18-year-old smoker who presents with 10-day history of sore throat, right otalgia, dysphagia to liquids and tender swelling in the right side of the neck.

A

G Correct answer

Scenario 4

A 12-month-old girl who presents with an expanding neck mass, present since birth, now causing dysphagia and airway difficulties.

A

F Correct answer

Scenario 5

An 11-year-old boy with a midline swelling which rises on tongue protrusion.

A Correct answer

Scenario 6

A 54-year-old smoker who presents with a lump in the left side of the neck and enlarged left tonsil.

A

D Correct answer

Scenario 7

A 35-year-old heavy smoker who presents with bilateral sore throat, symmetrically enlarged tonsils and bilateral neck swellings.

A

Page 7: 51 100

I Correct answer

THEME: facial pain

A Temporo-mandibular joint (TMJ) dysfunction

B Acute sinusitis

C Trigeminal neuralgia

D Dental abscess

E Acute parotiditis

F Migraine

G Temporal arteritis

H Acute otitis media

For the scenarios below, choose the most likely diagnosis from the list above. Each diagnosis may be used once, more than once or not at all.

Diagnosing facial pain can be problematic so it is essential to understand pathological causes for these conditions. The commonest cause of otological pain without ear involvement is TMJ dysfunction, a raised ESR should give suspicion of temporal arteritis and migraine is usually associated with a myriad of symptoms. Sinusitis and dental problems are commonly confused, however the former is very unlikely without nasal symptoms.

Scenario 1

Constant otalgia with a clicking jaw.

A Correct answer

Scenario 2

Pain and facial swelling only when eating.

A

Page 8: 51 100

E Correct answer

Scenario 3

Bi-temporal headache and raised erythrocyte sedimentation rate (ESR).

A

G Correct answer

Scenario 4

Sharp pain across the face with wind exposure.

A

C Correct answer

Scenario 5

Frontal headaches, dizziness and photophobia.

A

F Correct answer

Scenario 6

Acute cheek swelling and erythaema, no nasal symptoms.

A

D Correct answer

THEME: Sterilisation, disinfection and antiseptics

A AutoclaveB Ethylene oxideC Ionising radiationD Glutaraldehyde

Page 9: 51 100

E Povidone-iodine

From the list above, choose the most appropriate agent to achieve sterilisation, disinfection or antisepsis for the following situations. Each answer may be used once, several times or not at all.

 

Scenario 1

Surgical instruments (eg forceps, scissors)

A Correct answer

An autoclave is a method utilising steam under pressure to sterilise surgical instruments and drapes. It destroys micro-organisms, viruses and spores.

Scenario 2

Endoscopes (eg gastroscope)

A

D Correct answer

Glutaraldehyde is a disinfecting agent. Its use does not ensure destruction of all micro-organisms and spores. It is used for disinfecting flexible endoscopes.

Scenario 3

Suture materials

A

B Correct answer

Ionising radiation is also used by manufacturers for sterilising disposable products, instruments and sutures but for smaller items such as sutures ethylene oxide is the most sensible option.

Scenario 4

Page 10: 51 100

‘Scrub up’ prior to surgery

A

E Correct answer

Povidone-iodine is an antiseptic used for skin preparation during ‘scabbing up’ prior to surgery.

Scenario 5

Surgical drapes

A Correct answer

An autoclave is a method utilising steam under pressure to sterilise surgical instruments and drapes. It destroys micro-organisms, viruses and spores.

Scenario 6

Urinary catheter

A

C Correct answer

Ionising radiation is used by manufacturers for sterilising disposable products, instruments and sutures

THEME: Chemotherapeutic agents A Bleomycin B Chlorambucil C Cisplatin D Cyclophosphamide E Docetaxel F Doxorubicin G Etoposide H 5-FluorouracilI FlutamideJ Goserelin acetateK Methotrexate

Page 11: 51 100

L MitozantroneM OxaliplatinN TamoxifenO Vincristine The following are descriptions of chemotherapeutic agents used in surgical oncology. Please select the most appropriate agent from the above list. The items may be used once, more than once, or not at all.

 

 

Scenario 1

A cytotoxic alkylating agent used to synergistically down-size liver metastases secondary to colorectal carcinoma.

A

M Correct answer

M – Oxaliplatin

Oxaliplatin belongs to a new class of platinum agents. It contains a platinum atom complexed with oxalate and diaminocyclohexane (DACH). The exact mechanism of action of oxaliplatin is not known. Oxaliplatin forms reactive platinum complexes, which are believed to inhibit DNA synthesis by forming inter-strand and intra-strand cross-linking of DNA molecules. Pre-clinical studies have shown oxaliplatin to be synergistic with 5-fluorouracil.

Scenario 2

The most widely used alkylating agent used in combination regimens for adjuvant chemotherapeutic treatment of nodepositive, early-stage breast cancer.

A

D Correct answer

D – Cyclophosphamide

Cyclophosphamide, an alkylating agent, is consistently used in all combination chemotherapy regimens. Combination regimens are superior to single agents in the

Page 12: 51 100

adjuvant setting. The Oxford Overview analysis and other large randomised trials have shown slight, but statistically significant, superiority of anthracycline-containing regimens (eg doxorubicin, an antitumour antibiotic, and cyclophosphamide) over traditional CMF (cyclophosphamide, methotrexate and 5-fluorouracil – the latter two are anti-metabolites). Adjuvant chemotherapy results in an approximately 25% decrease of breast cancer mortality. However, the determination of the anthracycline-containing regimen of choice is still under investigation. Cyclophosphamide causes prevention of cell division primarily by crosslinking DNA strands. It is cell-cycle phase non-specific.

Scenario 3

A luteinising hormone-releasing hormone agonist used in T1–2 N0 M0 prostate cancer

A

J Correct answer

J – Goserelin acetate

Goserelin acetate is a luteinising hormone-releasing hormone (LHRH) agonist (unlike flutamide which is an androgen antagonist). Non-surgical management of prostate cancer involves a programme of regular clinical (with rectal) examination and prostatic specific antigen monitoring. It is considered in patients of advanced age or those who have significant lifelimiting co-morbidities and a life expectancy of less than 15 years. Androgen ablation is used in situations in which patients are unwilling to undergo potentially curative treatment options yet want some form of treatment above watchful waiting. Androgen ablation can be performed in multiple ways, such as using LHRH agonists and antagonists or oral antiandrogens (steroidal and non-steroidal).

Scenario 4

The basis of nearly all regimens of adjuvant and palliative treatment of colorectal cancer.

A

H Correct answer

H – 5-Fluorouracil

This anti-metabolite is usually used in combination with folinic acid (Leucovorin). There is very good evidence now for its use in the adjuvant setting for Dukes C tumours and some evidence for use in high-risk Dukes B tumours. Regimens may also

Page 13: 51 100

include irinotecan and oxaliplatin. It now exists in an oral form (Capecitabine), which is expensive but obviously advantageous.

THEME: On-table positionsA Armchair B Lateral C Lateral decubitus D Lithotomy E Lloyd Davies F ProneG Prone jack-knifeH Reverse TrendelenbergI SupineJ Trendelenberg

Above is a list of positions that are employed for various surgical procedures. From the operations described below please choose the most suitable on-table position. The items may be used once, more than once, or not at all.

 

Scenario 1

A 60-year-old man undergoing abdomino-perineal excision of the rectum.

A

E Correct answer

E – Lloyd Davies

The patient lies supine on the table, with legs in supports that flex the hips and knees to 45°. The legs can then be separated to allow surgical access to both abdomen and perineum at the same time (as is required during an abdomino-perineal excision of the rectum). To access the pelvis the patient is often also tilted head down, ie Trendelenberg. The lithotomy position is a more exaggerated version of the Lloyd Davies, where hips and knees are flexed to 90°. The lithotomy position was named after the operation that it was historically invented for; the removal of bladder stones. ‘Cutting for stone’ or lithotomy (lithos = stone) was frequently performed by travelling surgeons before the advent of anaesthesia and antisepsis (or fellowships).

Scenario 2

Page 14: 51 100

A 42-year-old woman undergoing long saphenous vein high ligation, stripping and avulsions.

A

J Correct answer

J – Trendelenberg

In the Trendelenberg position the patient is placed supine, with head-down tilt. This is the most appropriate patient placement for varicose vein surgery as it helps to alleviate pressure in the lower limb venous system and hence decreases intra-operative blood loss. This position can be also be used during laparoscopic pelvic surgery (eg gynaecological intervention, inguinal hernia repair, rectopexy) to keep bowel loops out of the operating field. The reverse Trendelenberg, as it is logically described, adopts a headup tilt. It is good for use in laparoscopic cholecystectomy/Nissen’s fundoplication, where the abdominal contents need to fall away from the region of intervention.

Scenario 3

A 64-year-old woman requiring a right-sided nephrectomy for renal cell carcinoma.

A

C Correct answer

C – Lateral decubitus

In the lateral decubitus position the patient is positioned on the contralateral side to their pathology, and the table is flexed in the centre. This stretches the flank of the patient that is uppermost, ie the side of the renal tumour. In this way there is better exposure of the loin between the bony prominences of the ribs and the iliac crest, clearly improving access and operating manoeuvrability.

Scenario 4

A 35-year-old man having an arthroscopic rotator cuff repair.

A Correct answer

A – Armchair

The seated armchair position is ideal for access to the shoulder, particularly in arthroscopic cases where dependency of the upper limb opens the subacromial space.

Page 15: 51 100

This allows for easier insertion of scope and instruments. In difficult cases it also permits a longitudinal or transverse incision through the deltoid to more fully open up the shoulder joint.

THEME: Side-effects of treatment for inflammatory bowel disease

A Azathioprine B Corticosteroids C MethotrexateD MetronidazoleE Sulfasalazine

For each of the options listed below, select the most likely associated side-effects from the list of drugs above. Each option may be used once, more than once, or not at all.

As well as nausea and vomiting, metronidazole may cause peripheral neuropathy, convulsions, headaches and hepatitis. Metronidazole is associated with irreversible peripheral neuropathy and does not appear to be related to the dose or length of time of ingestion. Hepatic fibrosis and pneumonitis from methotrexate use is related to the total dose given. Other adverse effects include: myelosuppression, nausea and vomiting, stomatitis, diarrhoea, osteoporosis, renal dysfunction, acute vasculitis (if high doses used) and seizures (especially after intrathecal use). Osteoporosis and cataracts are well-known complications of corticosteroids. Susceptibility to infection, Cushingoid appearance, central obesity, buffalo hump, moon facies, diabetes, hypertension, peptic ulceration, thin and easily bruised skin, striae, mental changes and proximal myopathy are associated side-effects.

Scenario 1

Irreversible peripheral neuropathy

A

D Correct answer

D – Metronidazole

Scenario 2

Page 16: 51 100

Reversible infertility due to oligospermia

A

E Correct answer

E – Sulfasalazine

Scenario 3

Osteoporosis

A

B Correct answer

B – Corticosteroids

Scenario 4

Cataracts

A

B Correct answer

B – Corticosteroids

Scenario 5

Hepatic fibrosis

A

C Correct answer

C – Methotrexate

Scenario 6

Page 17: 51 100

Pneumonitis

A

C Correct answer

C – Methotrexate

THEME: Surgical investigations

A Abdominal CT scan B Abdominal ultrasound C Barium enema (double contrast) D ColonoscopyE Endoscopic retrograde cholangiopancreatography (ERCP)F Gastrografin enemaG Small bowel follow-throughH Capsule endoscopy

For each of the following scenarios, select the most appropriate investigation from the above list. Each option may be used once, more than once, or not at all.

No technique is certain to reveal a colovesical fistula, but cystoscopy and barium enema are the two most likely investigations to help (one may see air in the bladder). X-ray of spun urine following a barium enema may also reveal barium radio-opacity. Pancreatic necrosis is not well visualised on ultrasound. Dynamic CT scans will usually demonstrate non-perfused pancreatic tissue.

Scenario 1

Psoas abscess

A Correct answer

A – Abdominal CT scan

Scenario 2

Small bowel tumour

A

Page 18: 51 100

G Correct answer

G – Small bowel follow-through

Scenario 3

Colovesical fistula

A

C Correct answer

C – Barium enema (double contrast)

Scenario 4

Pancreatic necrosis

A Correct answer

A – Abdominal CT scan

THEME: Complications following kidney transplantation

A Acute graft rejectionB Chronic graft rejectionC Delayed graft functionD LymphoceleE Renal artery stenosisF Ureteric obstruction

For each of the clinical scenarios listed below, select the most likely complication. Each option may be used once, more than once, or not at all.

 

Scenario 1

6 months following transplantation a patient is seen in clinic with a mild swelling and tenderness in the region of the transplant. Their renal function has deteriorated. A

Page 19: 51 100

computed tomography (CT) scan demonstrates a collection between the lower pole of the kidney and bladder.

A

D Correct answer

Leakage from perivascular lymphatic vessels can lead to significant collections of lymph between the lower pole of the transplanted kidney and the bladder. A lymphocele can present as pain, swelling and impaired renal function within the first year following transplantation. Treatment is by drainage. Percutaneous drainage has a high infection risk and is rarely successful, due to a high recurrence rate. Laparoscopic drainage of the collection in to the abdominal cavity is the treatment of choice.

Scenario 2

You are referred a patient by their general practitioner (GP). They underwent a renal transplantation 3 years ago. The patient has been complaining of headaches and dizzy spells. The GP reports an abrupt onset of unexplained hypertension.

A

E Correct answer

Renal artery stenosis complicates 2–10% of renal transplants. It may be confirmed on Doppler ultrasound. It can be managed using percutaneous angioplasty and stent placement.

Scenario 3

You review a 50-year-old man in clinic. He had a renal transplantation 200 days ago. He feels tired but otherwise well. His renal function is deteriorating and he has developed proteinuria.

A

B Correct answer

This gentleman has chronic allograft nephropathy. It is characterised by slowly progressive graft dysfunction which leads to chronic renal failure. In addition many patients develop nephrotic-range proteinuria. Histopathological examination of a renal allograft biopsy may show varying combinations of lesions. These include: chronic transplant glomerulopathy, ischaemic glomerulopathy, interstitial fibrosis, tubular atrophy, arteriosclerosis and arteriolar hyalinosis.

Page 20: 51 100

THEME: Postoperative management and critical care

A AtelectasisB Chest infectionC Anastomotic leakD Urinary tract infectionE Pulmonary embolism

The following patients develop a pyrexia postoperatively. What is the most likely cause in each scenario?

Postoperative pyrexia is common. The timing of the pyrexia often gives a clue to its cause:

24–72 h: atelectasis, chest infection 3–7 days: chest infection, wound infection, urinary

tract infection, anastomatic leak 7–10 days: deep vein thrombosis, pulmonary embolism.

Scenario 1

A 65-year-old gentleman who is day one post-left hemicolectomy for cancer of the bowel. On examination, he is lying flat in bed, does not have productive sputum and is complaining of abdominal pain which is preventing him from breathing in deeply.

A Correct answer

A – Atelectasis

Scenario 2

A 70-year-old gentleman who develops a productive, green cough 4 days post-left hemicolectomy for bowel cancer. He is a previous heavy smoker. Examination shows deceased air entry at the right lung base, with a few scattered crepitations

A

B Correct answer

B – Chest infection

Scenario 3

Page 21: 51 100

A 63-year-old gentleman 4 days post-left hemicolectomy for bowel cancer. He complains of vague abdominal pain in the suprapubic region, has passed very little flatus and no stool yet. Bowel sounds are absent.

A

C Correct answer

C – Anastomotic leak

Scenario 4

A 75-year-old smoker develops shortness of breath 10 days post-left hemicolectomy for bowel cancer. On examination he has a respiratory rate of 25/min and decreased air entry to the right lung base. Postoperative mobility has been slow because of osteoarthritis of his left hip.

A

E Correct answer

E – Pulmonary embolism

16 THEME: Head injury

A Cerebral contusion

B Diffuse axonal injury

C Extradural haematoma

D Cerebral infarction

E Scalp haematoma

F Subarachnoid haemorrhage

G Subdural haematoma

For each of the clinical scenarios listed below, select the most likely type of injury sustained. Each option may be used once, more than once, or not at all.

 

Scenario 1

Page 22: 51 100

A 4-year-old boy is brought to the emergency department by his mother. She witnessed him running into a table, hitting his head. There was no loss of consciousness and he cried immediately. He now has a headache and a large swelling on his forehead. She is concerned about a skull fracture.

A

E Correct answer

The child did not lose consciousness and has no neurological signs. The swelling is a scalp haematoma. With the mechanism of injury and subsequent history it is very unlikely that this child will have a fractured skull.

Scenario 2

A 41-year-old women is brought in by ambulance. She had complained of a very severe headache and then collapsed. On arrival she is confused and unable to give a coherent history.

A

F Correct answer

This is a classic history of a ruptured Berry aneurysm.

Scenario 3

An 80-year-old is referred to you following a fall. The residential home reports that he has had a number of falls recently and is becoming increasingly confused. He takes warfarin for his atrial fibrillation but is otherwise well.

A

G Correct answer

Chronic subdural haematomas can have an insidious onset. This gentleman is on warfarin, which is a risk factor.

Scenario 4

Page 23: 51 100

You are asked to review a 23-year-old man who has been assaulted with a baseball bat. He was found unconscious but now has a Glasgow Coma Scale of 14. He has had a computerised tomography (CT) scan of his brain which shows a biconvex hyperdense area in the right temporal region.

A

C Correct answer

Traumatic extradural haematomas are typically biconvex or lenticular in shape on computerised tomography (CT).

THEME: SHOCK

A CardiogenicB SepticC HypovolaemicD AnaphylacticE Obstructive

 

Scenario 1

A patient developed acute renal failure 15 d post quadruple artery bypass operation with a complicated post-operative course. A central line has been inserted. The patient has been hypotensive for the last 2 h; this is improving with fluid resuscitation. He develops distended jugular venous pressure, a sudden drop in blood pressure, and right-sided chest pain.

B

E Correct answer

Distended neck veins usually indicate obstructive shock; in this case, it is likely to be due to an iatrogenic pneumothorax when the central line was placed. This patient has also been hypovolaemic, but the most pressing problem is the pneumothorax.

Scenario 2

A 28-year-old motor cycle dare devil attempted to jump over 21 cars. He hit a wall sideways and sustained multiple long bone fractures and a deceleration injury. He is clinically shocked with angiographic evidence of a ruptured aorta.

A

Page 24: 51 100

C Correct answer

This is a clear case of hypotension; the patient has multiple fractures and a ruptured aorta; the injury could also lead to obstructive shock; but from the details given, hypovolaemic shock is by far the most likely diagnosis.

Scenario 3

A 55-year-old man has been in a significant crush injury involving a forklift truck. He has fractured his sternum as well as multiple ribs. He has quiet heart sounds and is peripherally poorly perfused with a raised jugular venous pressure.

A

E Correct answer

Quiet heard sounds and a raised jugular venous pressure indicate cardiac tamponade. This is an obstructive shock, and the blood needs to be evacuated urgently, either by pericardiocentesis or surgically.

Scenario 4

A 68-year-old woman commences a blood transfusion as she has lost a lot of blood post-operatively following neck dissection for a base-of-tongue carcinoma. She has been adequately resuscitated, but her blood pressure falls suddenly. She becomes tachycardic and short of breath, and feels peripherally warm.

A

D Correct answer

There is a correlation between the blood transfusion and fall in blood pressure. She has had an anaphylactic reaction to the blood, the commonest cause being ly labelled blood.

Scenario 5

A 24-year-old teacher with Crohn disease has unfortunately developed an enterocutaneous fistula post-bowel resection 2 weeks ago. She has been stable on the ward and managing the fistula well. Her blood results are all normal but show an upward trend of urea and creatinine.

Page 25: 51 100

A

C Correct answer

Hypovolaemic shock can also be secondary to high-output fistulas.

THEME: Spinal injuries A Cauda equina syndromeB Neurogenic shockC Cord transection at the C5 levelD ‘Hangman’s fracture’E C2 odontoid dens dislocationF A C1 ‘Jefferson fracture’ G Lumbar wedge fracture 

Match the statements below with the most appropriate spinal injury listed above. Each option may be used once only, more than once or not at all.

A ‘Jefferson fracture’ is caused by axial compression typically from a head injury and results in a blow out fracture of the C1 ring. A ‘Hangman’s fracture’ involves the posterior elements of C2 and is usually caused by distraction, so should not be treated by traction. A cord injury at the C5 level will result in flaccid arreflexia, diaphragmatic breathing, and ability to flex but not extend at the elbow. The patient will grimace to pain above but not below the elbow. Priapism may be present.A fracture in the thoracolumbar region may result in injury to the nerve roots that comprise the cauda equina as the spinal cord terminates at this level. Clinically there will be bladder and bowel signs and patchy sensory and motor losses in the lower limbs.Neurogenic shock is caused by the loss of sympathetic outflow after a cord injury. Lack of vasomotor tone results in pooling of blood in the extremities and a bradycardia. Careful use of vasopressors may be indicated.

Scenario 1

Caused by axial loading in a head injury leading to a ‘blow out’ fracture

B

F Correct answer

Page 26: 51 100

A C1 'Jefferson fracture'

Scenario 2

Diaphragmatic breathing will be present

A

C Correct answer

Cord transection at the C5 level

Scenario 3

Can be detected on an X-ray by applying Steel’s rule of three

A

E Correct answer

C2 odontoid dens dislocation

Scenario 4

May require the use of vasopressors

A

B Correct answer

Neurogenic shock

Scenario 5

Will result in bladder signs and patchy motor weakness in the lower limbs

A Correct answer

Cauda equina syndrome

Page 27: 51 100

Scenario 6

Skull traction is contraindicated

A

D Correct answer

‘Hangman’s fracture’

Scenario 7

Bradycardia will be present

A

B Correct answer

Neurogenic shock

Scenario 8

Flexion of the arms is preserved but extension is absent

A

C Correct answer

Cord transection at the C5 level

Scenario 9

Grimaces to pain above, but not below the clavicle

A

C Correct answer

Cord transection at the C5 level

Page 28: 51 100

THEME: Complications of gallstone disease

A Acute cholecystitisB Acute pancreatitisC Ascending cholangitisD Biliary colicE Chronic cholecystitisF Empyema of the gallbladderG Gallbladder perforationH Gallstone ileusI Mirizzi’s syndromeJ MucocoeleK Obstructive jaundice

The following are descriptions of patients with complications of gallstone disease. Please select the most appropriate diagnosis from the above list. The items may be used once, more than once, or not at all.

When considering this subject, the candidate is advised to think of complications within and without the gallbladder, starting with the former and progressing to the ileum.

Scenario 1

A 72-year-old woman presents with severe colicky central abdominal pain and vomiting. On examination there is right upper quadrant tenderness, negative Murphy’s sign, abdominal distension and tinkling bowel sounds. Blood results: total bilirubin 12 µmol/litre, aspartate aminotransferase 25 iu/litre, alanine aminotransferase 29 iu/litre, alkaline phosphatase 200 iu/litre, amylase 38 iu/litre; white cell count 8.0 x 109/litre.

B

H Correct answer

H – Gallstone ileus

This patient has symptoms typical of small bowel obstruction. Small bowel obstruction from impaction of a gallstone in the distal ileum is rare but most commonly occurs in women over 70 years old. It accounts for 20% of older patients with small bowel obstruction who do not have a history of a hernia or previous abdominal surgery. Plain abdominal films may show a classic triad of small bowel obstruction, a gallstone in the gut and gas in the biliary tree (although the Editor has yet to observe this in practice). Treatment is surgical with proximal enterotomy, removal of the stone, a search for further stones but not cholecystectomy (which is

Page 29: 51 100

dangerous and unnecessary). A bonus mark could be given for this question for also choosing Mirizzi’s syndrome because most patients also have a fistula.

Scenario 2

A 63-year-old man presents with a 5-day history of severe upper abdominal pain and vomiting. On examination there is a profound tachypnoea and tachycardia; he has generalised upper abdominal tenderness.

C

B Correct answer

B – Acute pancreatitis

The differential diagnosis is gallbladder perforation but this is much less common than pancreatitis. Typically patients present with severe upper abdominal pain radiating to the back, often associated with nausea and vomiting. Signs of cardiovascular or respiratory dysfunction may be present, abdominal signs range from mild epigastric or left upper quadrant tenderness to generalised peritonitis.

THEME: Benign and malignant pigmented skin lesions

A EphelisB Nodular melanomaC Giant congenital naevusD Lentigo maligna melanomaE Blue naevusF Dysplastic naeviG Solar lentigoH Amelanotic melanomaI Acral lentiginous melanoma

Select the most likely diagnosis for each of the patients below.

 

Scenario 1

A 32-year old woman presents to her general practitioner with a raised, polypoidal and deeply pigmented lesion over her right knee. It is sharply delineated from the surrounding skin. She says that it feels itchy and it bleeds occasionally.

B Correct answer

Page 30: 51 100

Nodular melanoma is the second most common subtype of all melanomas. These may occur over any part of the body although they are more common over the legs and trunk. These are frequently raised, palpable, deeply pigmented, and usually convex in shape; they may bleed or ulcerate. They are well demarcated from the surrounding skin. Lymphatic involvement occurs early. This type of melanoma has the worst prognosis among all forms of melanomas.

Scenario 2

A 72-year-old farmer presents to his general practitioner with an irregular, brown pigmented lesion over his right cheek. He says that he has had a brown patch over this area for nearly 10 years but this has recently got bigger. On examination, the patch feels thick and there is a discrete nodule within the lesion.

A

D Correct answer

Lentigo maligna melanoma (Hutchinson’s melanotic freckle) almost always arises over the sun-damaged skin of the face, neck, and arms and legs of elderly individuals. It is the least malignant variety and it presents as an irregular brown patch. The precursor in situ lesion, lentigo maligna, is usually present for more than 10–15 years and attains large size (4–8 cm diameter) before progressing to malignancy. Malignant degeneration is characterised by thickening and the development of discrete tumour nodule within the lesion.

Scenario 3

A 75-year-old lady present to her general practitioner with an erythematous, irregular papule over the sole of her foot. It does not appear pigmented. On examination, lymph nodes are palpable in the inguinal region.

A

H Correct answer

Amelanotic melanoma is a variant of melanoma in which the cells do not make melanin. Hence it does not appear darkly pigmented. It may, however, occasionally appear light brown or tan with grey edges. Classically, the lesions are pink or red, appearing as erythematous papules or nodules. They have irregular edges/margins. Amelanotic melanoma may occur in any one of the four clinico-pathological variants (lentigo maligna melanoma, superficial spreading melanoma, nodular melanoma and acral-lentiginous melanoma). Patients with amelanotic melanoma frequently present with advanced disease and lymph node

Page 31: 51 100

involvement. Their prognosis is poor, probably because of their delayed presentation and/or diagnosis.

THEME: Pancreatic diseases

A Pancreatic pseudocystB Carcinoma head of pancreasC Pancreatic fistulaD InsulinomaE Chronic pancreatitisF Carcinoma tail of pancreasG Gastrinoma

For each of the following patients, select the most likely diagnosis from the above list. Each option may be used once, more than once, or not at all.

Because of its close proximity to the tail of the pancreas, it is not uncommon for pancreatic fistulae to develop after splenic surgery. Whipple’s triad relates to overproduction of insulin. Courvoisier’s sign is a palpable gallbladder which does not occur with cholecystitis or gallstones.

Scenario 1

Obstructive jaundice, Courvoisier’s sign present

A

B Correct answer

Scenario 2

Plain abdominal X-ray shows calcification in the ducts

A

E Correct answer

Scenario 3

Computed tomography (CT) shows localised collection in the lesser sac

Page 32: 51 100

A Correct answer

Scenario 4

Post-splenectomy

A

C Correct answer

Scenario 5

Whipple’s triad of symptoms present

A

D Correct answer

THEME: Gastrointestinal secretions

A Parietal cellB G cellC -CellD Chief cellE -CellF Pancreatic acinar cellG Enterochromaffin cell

For each of the options below, choose the single most likely source from the list above. Each option may be used once, more than once, or not at all.

 

Scenario 1

Chymotrypsinogen

A

F Correct answer

Page 33: 51 100

Scenario 2

Pepsinogen

A

D Correct answer

Scenario 3

Serotonin

A

G Correct answer

Scenario 4

Gastrin

A

B Correct answer

Scenario 5

Insulin

A

E Correct answer

Scenario 6

Hydrochloric acid

A Correct answer

Page 34: 51 100

Scenario 7

Colipase

A

F Correct answer

Scenario 8

Glucagon

A

C Correct answer

Scenario 9

Amylase

A

F Correct answer

Scenario 10

Phospholipase

A

F Correct answer

THEME: Inhalation of foreign bodies.

Which segments of which lung would the foreign body come to lie in.

A Left apicalB Right apicalC Right anterior

Page 35: 51 100

D Right anterior basalE Right lateral basal

The main bronchopulmonary segments are as followsfor the right lung

        superior lobe – (1) apical, (2) posterior, (3) anterior        middle lobe – (4) lateral, (5) medial        inferior lobe – (6) superior (apical), (7) medial basal, (8) anterior basal, (9) lateral basal, (10) posterior basal

for the left lung        superior lobe – (1) apical, (2) posterior, (3) anterior, (4) superior lingular, (5) inferior lingular        inferior lobe – (6) superior (apical), (7) medial basal, (8) anterior basal, (9) lateral basal, (10) posterior basal.

Since the right bronchus is the wider, and more direct, continuation of the trachea, foreign bodies tend to enter the right bronchus, from then they usually pass into the middle or lower lobe bronchi.

Scenario 1

A 12-year-old footballer is upright when he inhales a tooth.

A

D Correct answer

Scenario 2

An 85-year-old man inhales his medication following a stroke.

A

E Correct answer

THEME: jaundice

A Gilbert's syndromeB Common bile duct stoneC Acute alcoholic hepatitisD Carcinoma of head of pancreasE Primary biliary cirrhosisF Ascending cholangitisG Hepatic metastasesH Cholangiocarcinoma

Page 36: 51 100

I Primary sclerosing cholangitis

For each of the patients described below, select the single most likely diagnosis from the options listed above. Each option may be used once, more than once or not at all.

 

Scenario 1

A 30-year-old man with a 7-year history of ulcerative colitis complains of fluctuating jaundice, pruritus, right upper quadrant pain and weight loss for the previous 6 months. Plasma alkaline phosphatase is 270 U/l (normal 30–130 U/l).

He is pyrexial with rigors.

A

F Correct answer

Ulcerative colitis is associated with cholangitis. Charcot’s triad in cholangitis is: right upper quadrant pain, jaundice and rigors. Treatment is with intravenous antibiotics.

Scenario 2

A 25-year-old man, who has noticed mild intermittent jaundice for the past 6 years becomes more icteric during a chest infection. There is a moderate unconjugated hyperbilirubinaemia, otherwise he has normal liver function tests and liver histology.

A Correct answer

Gilbert’s syndrome is an inherited metabolic disorder leading to increased unconjugated hyperbilirubinaemia. Jaundice usually occurs during intercurrent illness.

Scenario 3

A 57-year-old woman presents with a 5-week history of itching, pale stools, dark urine and fatigue. She has tender 12 cm hepatomegaly, splenomegaly, mild ascites, widespread spider naevi and palmar erythaema. Laboratory investigations show a leucocytosis of 20 109/l, plasma alanine transaminase 280 U/l (normal 2–50 U/l), plasma albumin 24 g/l (normal 35–50 g/l) and a prothrombin time of 20 seconds

Page 37: 51 100

(control 13 seconds). IgM is increased. She is positive for antimitochondrial antibodies.

A

E Correct answer

Primary biliary cirrhosis is a slowly progressive cholangiohepatitis. More common in women than men, peak age 45 years. Treatment is mainly symptomatic. Liver transplantation is offering encouraging results.

Scenario 4

A 62-year-old man presents with a 3-month history of weight loss, increasing jaundice, dyspepsia and nocturnal epigastric pain radiating to his back. He had an anterior resection of the rectum 5 years ago. There is a right upper quadrant mass, which moves on respiration. Laboratory investigations show plasma alkaline phosphatase of 752 U/l (normal 30–130 U/l) and 1% glycosuria.

A

D Correct answer

Carcinoma of the head of the pancreas usually present insidiously with weight loss, epigastric pain radiating to the back and features of obstructive jaundice. Only approximately 13% of patients at presentation are suitable for a Whipple’s procedure. Operative mortality is high at around 20%. Often palliative bypass surgery is all that can be done.

THEME: Drugs used in critical care A Adenosine B Adrenaline C Amiodarone D Amrinone E Atropine F Digoxin G DobutamineH DopamineI DopexamineJ LignocaineK NitroglycerinL Noradrenaline 

The following are descriptions of drugs used in the management of the critically ill patient. Please select the most appropriate drug

Page 38: 51 100

from the list. The items may be used once, more than once, or not at all.

 

Scenario 1

A predominant ß1-agonist, used to improve cardiac output in patients with myocardial failure, provided intravascular volume is satisfactory.

B

G Correct answer

G – Dobutamine

Dobutamine stimulates both ß1- and ß2-receptors. Stimulation of ß1-receptors produces a good cardiac inotropic and chronotropic response, leading to improved cardiac output, and stimulation of ß2-receptors produces a degree of vasodilatation, especially in skeletal muscle (‘inodilatation’). Dobutamine can be used in combination with noradrenaline if sepsis and hypotension are a problem. Studies have demonstrated that dobutamine is more effective than dopamine (dosage-dependent roles, for example at low doses it is a D1A-agonist, at intermediate doses ß1-adrenoreceptor effects appear, and at high doses a1-effects predominate) when improvements in oxygen delivery [D(O2)] and uptake [V(O2)] are considered.

Scenario 2

A phosphodiesterase III inhibitor; acting as both a positive inotrope and a peripheral vasodilator; it is effective in cardiogenic shock.

A

D Correct answer

D – Amrinone

Amrinone (and enoximone) are phosphodiesterase III inhibitors that increase intracellular cyclic AMP. They improve hypotension, principally caused by cardiogenic shock, by their dual action of increasing cardiac output and decreasing systemic vascular resistance (‘inodilatation’). The addition of dobutamine is considered to be synergistic.

Scenario 3

Page 39: 51 100

A predominant a1-agonist; the first line in patients with septic shock.

A

L Correct answer

L – Noradrenaline

Noradrenaline stimulates a1-adrenoreceptors with minor ß1- and ß2-effects. It is employed conventionally when increased systemic vascular resistance (to increase the blood pressure by increasing left ventricular after-load) is required to maintain the mean arterial pressure after fluid replacement and dobutamine infusion have proved inadequate. This is commonly the case in septic shock where inflammatory mediator activation causes systemic vasodilatation.

THEME: Pancreatic tumour

A Dumping syndromeB GlucagonomaC InsulinomaD VIPomaE Zollinger–Ellison syndrome

For each of the patients described below, select the most likely pancreatic tumour from the list of options above. Each option may be used once, more than once, or not at all.

 

Scenario 1

A vicar who missed breakfast swore during his sermon, but felt better after a late breakfast.

A

C Correct answer

C – Insulinoma

Insulinomas produce episodes of hypoglycaemia leading to altered behaviour and disturbances of consciousness. Characteristically, the patient feels well between episodes. It is a difficult diagnosis to make unless there is a degree of clinical suspicion.

Page 40: 51 100

Scenario 2

A 52-year-old man with hypercalcaemia suffers from recurrent gastric ulcers.

A

E Correct answer

E – Zollinger–Ellison syndrome

In the second case, the patient has MEN-1. The hypercalcaemia arises from hyperparathyroidism. He also has Zollinger–Ellison syndrome, which causes markedly raised levels of gastrin and gastric acid hypersecretion. This leads to severe ulceration not only in the stomach and duodenum but also the jejunum.

THEME : Premalignant lesions/cutaneous malignancies

A Bowen's diseaseB Xeroderma pigmentosumC Squamous cell carcinomaD Actinic keratosisE Basal cell carcinomaF Basal cell naevus syndromeG Marjolin's ulcerH Cutaneous malignant lymphomaI Keratoacanthoma

Select the most likely diagnosis for each of the patients below.

 

Scenario 1

A 75-year-old farmer presents with a painless, well-defined ulcer over the lateral aspect of his left angular region (lateral corner of the eye). On examination, it has a pearly rolled-out edge. He states that he noticed it eight months ago as a small, pearly nodule which soon ulcerated.

B

E Correct answer

Basal cell carcinoma (BCC) is the most common cutaneous malignancy. It may be single or multiple, and commonly occurs in the sun-exposed areas of the elderly. Nodular or the nodulo-ulcerative type is the commonest form of BCC. In

Page 41: 51 100

this type, the tumour is usually single, and commences as a small, waxy or pearly nodule with clearly defined margins. The pearly appearance is more apparent on lightly stretched skin and it may be covered with surface telangiectasia. As the tumour enlarges, central ulceration occurs, resulting in the characteristic rolled-out edge. BCC is frequently painless in the early stages; pain signifies deeper extension with underlying tissue involvement.

Scenario 2

A 64-year-old builder presents with a seven-week history of a rapidly growing, solitary, fleshy and dome-shaped nodule over his right nasolabial region. On examination, it has got a central hyperkeratotic core.

A

I Correct answer

Keratoacanthoma (molluscum sebaceum) is a cutaneous lesion commonly seen in sun-exposed sites. Although it is commonly thought to be benign, some authorities maintain that they are, in fact, well-differentiated squamous cell carcinomas (SCCs) rather than a distinct clinical entity; hence it is also known as the ‘self-healing’ SCC. These tumours may be solitary or multiple, and are more common in males. It presents as a fleshy, elevated and nodular lesion with an irregular crater shape, and has a characteristic central hyperkeratotic core. The most significant histological feature is its rapid growth (6–8 weeks). The short history and a rapid increase in size suggest keratoacanthoma rather than a SCC. In some cases, it undergoes spontaneous resolution within six months of onset, leaving behind a depressed scar.

Scenario 3

A 70-year-old ex-sailor presents with an exophytic ulcer over his left cheek. On examination, the ulcer has got an everted edge. A few enlarged lymph nodes are palpable in his neck.

A

C Correct answer

Squamous cell carcinoma (SCC) is seen primarily in older patients, mostly men. Like BCC, the prime aetiological factor is excessive exposure to solar radiation. Farmers, ranchers, sailors, and all those whose occupation requires excessive sun exposure are predisposed to SCC. In addition to radiation, chemicals, cytotoxic drugs, immunosuppressant drug treatment, chronic lesions and a wide variety of dermatoses also play a role in its development. There are two main types of SCC:

Page 42: 51 100

(i) a slow-growing variety that is verrucous in nature and exophytic in appearance. This is locally invasive, penetrating deeper structures, and is more likely to metastasize; and (ii) a nodular and indurated type, with rapid growth and early ulceration combined with local invasiveness. Metastasis is late compared to the verrucous type.

THEME: Paediatric investigations A A‘cone’ on contrast enemaB A ‘double-bubble sign’ on plain abdominal X-ray (AXR)C A ‘target’ lesion on abdominal ultrasoundD A type-II curve on a diuretic renogramE Air-filled cysts in the left chestF Clubbed renal calyces on a micturating cystogramG Hypochloraemic metabolic alkalosisH Intramural gas on plain AXR 

For each of the clinical scenarios below, select the most appropriate feature on investigation from the above list. Each option may be used once, more than once, or not at all.

 

Scenario 1

A 10-hour-old term baby presents with persistent bile-stained vomiting.

A

B Correct answer

B – A ‘double-bubble sign’ on plain AXR

Persistent bile-stained vomiting (if the atresia is distal to the second part where the common bile duct enters the duodenum) in a newborn infant may indicate a diagnosis of duodenal atresia. The typical radiological feature is the ‘double-bubble’ seen on plain AXR (an air-filled dilated stomach and first part of duodenum, creating two discrete air shadows).

Scenario 2

A 10-day-old baby, born at 30 weeks, presents with bile-stained vomiting and bloody diarrhoea.

Page 43: 51 100

A

H Correct answer

H – Intramural gas on plain AXR

A premature baby presenting with bile-stained vomiting and rectal bleeding is at high risk of suffering from necrotising enterocolitis. The typical radiological feature of this condition is gas within the bowel wall on plain AXR. It may be associated with gas in the portal system and/or free air in the abdomen when the disease progresses to intestinal perforation.

Scenario 3

An 8-month-old infant presents with colicky abdominal pain and bleeding per rectum.

A

C Correct answer

C – A ‘target’ lesion on abdominal ultrasound

An 8-month-old infant with colicky abdominal pain and rectal bleeding is likely to have intussusception, which may be diagnosed on abdominal ultrasound by the presence of a ‘target lesion’ or by gastrografin enema. The ‘target’ lesion represents the layers of the bowel, one invaginated inside the other, seen in transverse section.

Scenario 4

A 2-day-old baby presents with abdominal distension, bile-stained vomiting and failure to pass meconium.

A Correct answer

A – A ‘cone’ on contrast enema

Hirschsprung’s disease typically presents in the first few days of life with abdominal distension, bile-stained vomiting and failure to pass meconium. The diagnosis can easily be made by the presence of a cone between collapsed distal (usually rectum) and proximal dilated bowel on a contrast enema. The cone represents the transition zone between normal proximal ganglionated bowel and distal aganglionic bowel. Confirmation of Hirschsprung’s disease, however, can only be made on histological biopsy.

THEME: Lower limb ulceration

Page 44: 51 100

A Basal cell carcinoma B Erythema nodosum C Hypertensive ulcer (Martorell’s ulcer) D Necrobiosis lipoidica E Neuropathic ulcerF Pyoderma gangrenosumG Sickle cell diseaseH Squamous cell carcinomaI Vasculitic ulcerJ Venous ulcer

For each of the following profiles, select the most likely cause of the ulcer from the above list. Each option may be used once, more than once, or not at all.

 

Scenario 1

A 54-year-old lady with known inflammatory bowel disease presents with a large nodulo-pustular ulcerating lesion over her right anterior shin. It has a blue overhanging necrotic edge. The arterial blood pressure index (ABPI) is normal.

A

F Correct answer

F – Pyoderma gangrenosum

Pyoderma gangrenosum is recurring nodulo-pustular ulceration that commonly affect the legs, abdomen and face. The ulcers are tender with a red-blue overhanging necrotic edge and they heal with cribriform scars. Pyoderma gangrenosum is associated with inflammatory bowel disease, acute leukaemia, polycythaemia rubra vera, autoimmune hepatitis, Wegener’s granulomatosis and myeloma.

Scenario 2

A 57-year-old obese lady with varicose veins presents with a large ulcer over her left medial malleolus. This is associated with surrounding lipodermatosclerosis and eczema. The ABPI in this leg is 1.05.

A

Page 45: 51 100

J Correct answer

J – Venous ulcer

Venous ulcers may develop spontaneously or following a minor injury. Risk factors include past history of varicose veins and deep-venous thrombosis (DVT); all causes of varicose veins or DVT may therefore predispose to venous ulceration. Venous incompetence leads to a high venous pressure which causes pericapillary fibrin deposition, white cell activation and increased production of free radicals. All these factors contribute to skin breakdown, causing ulceration. These ulcers usually lie proximal to the medial or lateral malleolus, although they may extend to the ankle or dorsum of the foot. Lipodermatosclerosis frequently accompanies long-standing venous incompetence. Increased exudates leads to venous eczema, excoriation of the surrounding skin, itch and worsening of the ulcer.

Scenario 3

A 59-year-old man presents with ulceration over the distal tips of three toes in his left foot and over his right heel. He also complains of paraesthesia in both his feet. The ABPI is 1.02 on the right and 1.16 on the left. On neurological assessment, proprioception and vibration sense are reduced.

A

E Correct answer

E – Neuropathic ulcer

Neuropathic ulcers can result from peripheral sensory neuropathy secondary to diabetes mellitus. The other causes for altered sensory neuropathy, leading to neuropathic ulcers include: spinal cord injuries, spina bifida, tabes dorsalis, syringomyelia, alcohol abuse and leprosy. Neuropathic ulcers are common at the sites of pressure or repeated trauma. The usual sites in the lower limb are the head of the metatarsals, interdigital clefts, heel (calcaneum) and the lateral malleolus. In diabetic patients, the arterial blood pressure index (ABPI) may be falsely elevated, because of calcification of the vessel walls and medial sclerosis, even in those with major vessel disease. Neurological assessment of the foot and toes may reveal altered sensation including proprioception and two-point discrimination, reduced vibration sense and absent ankle jerks.

Scenario 4

A 30-year-old Afro-Caribbean lady presents with a 4-month history of a painful ulcer over her right shin. ABPIs are normal. She is anaemic and has mild splenomegaly.

Page 46: 51 100

A

G Correct answer

G – Sickle cell disease

Sickle cell disease is hereditary haemolytic anaemia occurring mainly among those of Afro-Caribbean origin. The haemoglobin ‘S’ molecule crystallises during reduced blood oxygen tension causing vascular occlusion. Depending on the affected vessel, patients may have bone or joint pain, priapism, neurological abnormalities, skin ulcers. Ulceration is due to occlusion of the cutaneous microcirculation, leading to ischaemia and skin breakdown. In dark-skinned individuals, there may be a non-specific increase in melanin pigmentation around the ulcer which, if over the ankle region, may be confused with haemosiderin discoloration around a venous ulcer.

THEME: Mediastinal conditions

A AchalasiaB Aortic dissectionC Carcinoma of the oesophagusD Mallory–Weiss tearE Marfan’s syndrome

For each of the scenarios described below, select the most likely diagnosis from the above list. Each option may be used once, more than once, or not at all.

Achalasia is the failure of the lower oesophageal sphincter to relax, with abnormal oesophageal peristalsis. It does not cause an anaemia unless there is malignant change.

Scenario 1

A tall, middle-aged woman presents complaining of a sudden onset of chest pain.

A

B Correct answer

B – Aortic dissection

Aortic dissection produces tearing chest and back pain together with neurological deficits from spinal cord and cerebral ischaemia.

Scenario 2

Page 47: 51 100

A 73-year-old man has anaemia, weight loss and difficulty swallowing.

A

C Correct answer

C – Carcinoma of the oesophagus

Oesophageal carcinoma is more common in elderly people and presents in advanced stages with progressive dysphagia.

Scenario 3

A young man is complaining of violent retching after an alcoholic binge.

A

D Correct answer

D – Mallory–Weiss tear

A Mallory–Weiss tear produces a linear mucosal tear in the lower oesophagus close to the gastro-oesophageal junction, which results in haematemesis and later melaena.

THEME: Cancer therapy options A Hormonal manipulationB Radiotherapy and steroidsC Surgical resectionD Systemic chemotherapy 

For each of the clinical scenarios listed below, select the most appropriate management options from the above list. Each option may be used once, more than once, or not at all.

 

Scenario 1

Recurrent non-Hodgkin’s lymphoma

Page 48: 51 100

A

D Correct answer

Prolonged remission, or even cure, can be achieved using systemic chemotherapy for recurrent non-Hodgkin’s lymphoma.

Scenario 2

Isolated pulmonary metastasis from colorectal cancer

A

C Correct answer

Isolated pulmonary metastasis from colorectal cancer (as occurs in 5% of patients) can be surgically removed with curative intent if the patient will tolerate a thoracotomy. It is only suitable if the primary tumour has been rigorously controlled and extrathoracic secondaries have been excluded.

Scenario 3

Residual anal squamous cell carcinoma (SCC) after local radiotherapy

A

C Correct answer

Residual anal squamous cell carcinoma post local radiotherapy is best treated by abdominoperineal resection with curative intent.

Scenario 4

Metastatic prostatic carcinoma not involving bone

A Correct answer

Metastatic prostatic carcinoma not involving bone is best treated by hormonal manipulation. If the bone is involved, radiotherapy is the treatment of choice, with or without internal fixation.

Scenario 5

Page 49: 51 100

Diffuse intracranial metastatic melanoma

A

B Correct answer

Multiple intracranial metastases are best treated with radiotherapy, whether over the whole brain or with stereotactic radiosurgery (gamma knife). Diffuse intracranial metastatic melanoma has an extremely poor prognosis. Short-term benefit may be achieved by radiotherapy with steroids; chemotherapy is of no benefit.

Testicular swellings

A Teratoma

B Seminoma

C Leydig-cell tumour

D Sertoli-cell tumour

For each of the following situations, select the most likely answer from the above list. Each option may be used once, more than once or not at all.

 

Scenario 1

A 22-year-old man presents with a painless swelling in his right testis.

A Correct answer

This is the commonest testicular tumour in this age group.

Scenario 2

A 25-year-old man presents with a painless swelling in his right testis and -fetoprotein (AFP) is raised.

A Correct answer

Page 50: 51 100

This is the commonest tumour in this age group and AFP is raised in 70% of these tumours.

THEME: Shock A Cardiogenic shockB Septic shockC Neurogenic shockD Hypovolaemic shock <15% volume lossE Hypovolaemic shock 15–30% volume lossF Hypovolaemic shock >40% volume loss 

For each of the patients described below choose the most appropriate variety of shock from the list above. Each may be used once, more than once or not at all.

 

Scenario 1

A motorcyclist was admitted to the emergency department having been thrown a distance of 30 feet. He was wearing a helmet and had recovered consciousness at the scene. He remained confused. Pulse 40 bpm, respiratory rate 30/min, blood pressure (BP) 75/60 mmHg. On catheterisation there was no urine output. Lateral cervical spine X-rays were suggestive of a fracture of C6.

A

C Correct answer

Neurogenic shock

Neurogenic shock is due to the loss of sympathetic tone and combines the symptoms characteristic of hypovolaemic shock with a profound bradycardia.

Scenario 2

A 25-year-old marathon runner was involved in a road traffic accident while out training. She was admitted to the emergency department with a pulse of 100 bpm, BP 75/60 mmHg, respiratory rate 30/min. Her abdomen was generally tender, peritoneal lavage was positive. There was no urine output.

A

Page 51: 51 100

F Correct answer

Hypovolaemic shock >40% volume loss

Scenario 3

A homeless gentleman was found collapsed in a dark alleyway at 5 am on New Year’s Day. He was unconscious and smelt of alcohol. On arrival in the emergency department his pulse was 110/min, BP 115/50 mmHg, he was apyrexial on admission, but his skin was noted to be flushed. During the secondary survey he was found to have sustained a penetrating abdominal injury, which looked a few days old.

A

B Correct answer

Septic shock

Generally in hypovolaemic shock <15% loss leads to anxiety, no change in pulse, blood pressure (BP) or respiratory rate, urine output is maintained; 15–30% loss leads to anxiety. The BP is maintained, the pulse generally 100 bpm and the pulse pressure reduced, urine output is maintained at 20–30 ml/h.; 40% loss causes confusion. The pulse is generally greater than 140/min, the BP and pulse pressure fall. There is no urine output. Fit athletic individuals may initially compensate for massive blood loss causing hypovolaemia maintaining pulse and BP, then rapidly decompensate. In cases of delayed presentation septic shock should be considered. This is similar in presentation to hypovolaemic shock but characterised by a wide pulse pressure. Patients who are hypothermic from exposure may initially appear to be apyrexial.

THEME: Head injury

A Base-of-skull fracture

B Extradural haemorrhage

C La Fort fracture type I

D La Fort fracture type II

E Subarachnoid haemorrhage

F Subdural haemorrhage

For each of the following scenarios choose from the above list the most likely diagnosis.

 

Page 52: 51 100

Scenario 1

A 69-year-old man, in sheltered accommodation, tripped over the edge of his coffee table 1 week ago. He didn’t lose consciousness. The warden says the patient has become increasingly confused over the last 3 weeks. On examination the patient smells strongly of alcohol and his body is covered with small bruises which seem to be the result of previous falls.

A

F Correct answer

Subdural bleeds often present a week or more after the initial injury, which may be relatively innocuous. Chronic alcoholics are at increased risk of having a subdural bleed; they are prone to recurrent falls, may have signs of chronic liver disease (spider naevi, bruising) and may have abnormal clotting factors that predisposes them to a higher risk of haemorrhage.

Scenario 2

A 23-year-old man presents to the casualty department after being punched in the face a few hours ago. There had been no loss of consciousness. He complains he cannot hear well out of his left ear. On examination, there appears to be blood behind the left tympanic membrane.

A Correct answer

A base-of-skull fracture should always be suspected if the patient has any of the following signs: bruising around the eyes (‘racoon eyes’), haemotympanum, CSF leak from the nose or ears, bruising behind the ear (Battle’s sign).

Scenario 3

A cricketer is hit by the ball on his left temporal region. He lost consciousness for a minute. Despite a small bruise just in front of his left ear tragus, he decides not to go to hospital because he says he feels fine. After one day, he develops a progressive headache and begins to vomit.

A

B Correct answer

Page 53: 51 100

This scenario is typical of someone with a possible extradural bleed. It is caused by a fracture of the temporal or parietal bone causing laceration of the middle meningeal artery. These patients may or may not suffer loss of consciousness at the time of the initial injury and appear fine (‘lucid interval’). They may deteriorate rapidly after a few hours/days complaining of symptoms associated with a rise in intracranial pressure (headache, vomiting, decrease in the level of consciousness).

Scenario 4

A 19-year-old is brought to A&E after being repeatedly punched in the face. On examination, his face is grossly swollen and there is a palpable step in his maxilla. There appears to be an intermittent discharge of clear fluid from his nose.

A

D Correct answer

Le Fort fractures lie between the frontal bone, base of the skull and mandible. They are classified as Le Fort I, II and III depending upon the severity of injury. Le Fort I involves only the tooth-bearing portion of the maxilla. Le Fort II involves the maxilla, nasal bones and medial aspects of the orbits. Le Fort III involves the maxilla, zygoma, nasal bones, ethmoid and the small bones of the base of the skull. Classically there is a ‘palpable step’ in the maxilla and there may be CSF rhinorrhoea, diplopia and conjuctival haematoma.

THEME: Hernias (types and taxonomy)A Femoral hernia B Gluteal hernia C Incarcerated hernia D Incisional hernia E Internal hernia F Littre’s hernia G Lumbar herniaH Maydl’s herniaI Obstructed herniaJ Obturator herniaK Richter’s herniaL Spigelian herniaM Strangulated hernia

The following patients all have hernias. From the list above, select the most appropriate diagnosis, according to the clinical and/or anatomical information provided. The items may be used once, more than once, or not at all.

Scenario 1

Page 54: 51 100

A 48-year-old gentleman presents with a 36-h history of vomiting, central abdominal pain and distension. On examination, he is unwell, febrile, dehydrated and tachycardic. Abdominal examination reveals a distended abdomen with ‘tinkling’ bowel sounds, and a very large right inguino-scrotal hernia that is irreducible, erythematous and tender. At operation, a W-loop of small bowel lies in the hernial sac.

A

H Correct answer

H – Maydl’s hernia

Maydl’s hernia is a complication of large hernial sacs, especially right scrotal hernias in Africans. It is characterised by a W-loop of small bowel lying in the sac, with strangulation of the ‘intervening’ loop within the main abdominal cavity by the constriction of the neck of the sac. The description of the operative findings differentiates this hernia from afferent loop strangulation (afferent loop entwined about afferent and efferent loops), a Richter’s hernia (part of the bowel at the anti-mesenteric margin becomes strangulated), and a Littre’s hernia (strangulation of a Meckel’s diverticulum). The operative findings also differentiate from ‘simple’ strangulation, which is associated with a similar clinical presentation, ie evidence of gut ischaemia (severe pain with systemic upset, eg fever, tachycardia) and obstruction (vomiting, and abdominal pain and distension).

Scenario 2

A 49-year-old woman is admitted with acute small bowel obstruction. She reports a 2-day history of severe pain radiating down the inner aspect of the right thigh to her knee. Consequently, you meticulously examine the hernial orifices in the groin, but no hernia is evident. Ultimately, she requires a laparotomy, as her obstruction fails to resolve with conservative management. Only then does the cause of her obstruction become apparent.

A

J Correct answer

J – Obturator hernia

Obturator hernias are six times more common in women than men, and three times more common after than before the age of 50 years. A preoperative diagnosis is rarely made, because a swelling is not always palpable in the thigh. Therefore, it is usually diagnosed during laparotomy for nonresolving small bowel obstruction, as in the case

Page 55: 51 100

described. Consequently, the operative mortality is approximately 30%. The peritoneum protrudes through the obturator canal, and then between the pectineus and abductor longus muscles to enter the femoral triangle. The Howship–Romberg sign of pain referred along the geniculate branch of the obturator nerve to the inner aspect of the knee should raise the suspicion of an obturator hernia. Other examples of hernias frequently only discovered during laparotomy for relief of intestinal obstruction include: gluteal and sciatic hernias (protruding through the greater and lesser sciatic notches, respectively), pelvic hernias (of the pouch of Douglas into the posterior wall of the vagina or vulva; not rectocoele or cystocoele, which are false hernias), and pudendal hernias (lateral protrusion of peritoneum through a persistent hiatus of Schwalbe between the origin of the levator ani from the obturator internus, usually following surgical removal of pelvic organs).

Scenario 3

A 65-year-old gentleman attends the outpatients department with a long history of an intermittent swelling in the right groin that is now persistent. The swelling is irreducible but is non-tender, and lies above and medial to the pubic tubercle. He denies any gastrointestinal disturbance.

A

C Correct answer

C – Incarcerated hernia

This question draws the candidates’ attention to the clinical differentiation of complications of hernias. Incarceration refers to fixation of contents within the hernia sac as a result of adhesions. Such a hernia is irreducible, but is neither tender, nor associated with systemic upset (differentiating it from a strangulated hernia), nor associated with gastrointestinal symptoms (differentiating it from an obstructed hernia).

THEME: Rectal bleeding

A Anal carcinomaB Anal fissureC AngiodysplasiaD Colonic carcinomaE Colonic polypF Crohn’s diseaseG Diverticular diseaseH HaemorrhoidsI Infective colitisJ Ischaemic colitisK Peri-anal haematomaL Peptic ulceration

Page 56: 51 100

M Ulcerative colitis

The following patients have all presented with rectal bleeding. Please select the most appropriate diagnosis from the above list. The items may be used once, more than once, or not at all.

 

Scenario 1

A 61-year-old renal transplant patient is referred to you on-call with acute-onset severe bloody diarrhoea. He appears clinically very unwell. He has no history of bowel problems.

A

I Correct answer

I – Infective colitis

Cytomegalovirus colitis can cause severe diarrhoea and torrential, even life-threatening, rectal bleeding. This diagnosis should always be considered first in patients on immunosuppression. This and other infections are common problems in acquired immune deficiency syndrome – other responsible organisms include herpes virus, and Cryptosporidium.

Scenario 2

A 27-year-old woman is seen with a 3-day history of acute diarrhoea which she attributes to food-poisoning. Today she has attended because of fresh rectal bleeding on the paper after wiping and once in the pan, separate from the stool.

A

H Correct answer

H – Haemorrhoids

Bright-red rectal bleeding in a young patient is invariably the result of haemorrhoids. Such bleeding is often triggered by trauma leading to ulceration of previously asymptomatic small piles. This can often be confirmed on proctoscopy in the acute phase but quite often you see the patient in outpatients weeks or months later when the problem has completely resolved. No further action need be taken.

Page 57: 51 100

Scenario 3

A 92-year-old woman presents with painless, bright-red rectal bleeding without other symptoms. Following a blood transfusion a barium enema is performed, the result of which is normal, and she is sent back to the nursing home. One week later, she rebleeds and returns to The Emergency Department. Again the bleeding settles, and after re-transfusion, she undergoes a gastroscopy and colonoscopy at which no abnormality is detected.

A

C Correct answer

C – Angiodysplasia

These are a type of arteriovenous malformation and are one of the common causes of significant lower gastrointestinal bleeding in the elderly population. As in this case, it is notoriously difficult to pinpoint the actual offending vessel. Where direct vision fails, mesenteric angiography or radionucleotide scans can sometimes be of diagnostic use but often also yield negative results if the vessel is not actively bleeding at the time of investigation. Should angiography demonstrate the source of bleeding, therapeutic embolisation can be performed. In cases of continued bleeding with negative investigations, treatment may involve total colectomy as a life-saving measure.

THEME: Mediastinal masses

A Anterior mediastinumB Middle mediastinumC Posterior mediastinumD Superior mediastinum

For each of the pathologies listed below, select the correct part of the mediastinum in which they are found from the above list. Each option may be used once, more than once, or not at all.

The locations of mediastinal masses include:

Anterior mediastinum: thymic lesions, lymphoma, germ-cell tumours, pleuropericardial cysts, lymph node enlargement.

Middle mediastinum: lymph node enlargement, bronchogenic cysts, enterogenic cysts

Page 58: 51 100

Posterior mediastinum: neural tumours, thoracic meningocele, oesophageal tumours, aortic aneurysms, paragangliomas

Superior mediastinum: thyroid masses, lymph node enlargement, oesophageal tumours, aortic aneurysms, parathyroid lesions

 

Scenario 1

Thymic lesions

A Correct answer

A – Anterior mediastinum

Scenario 2

Neural tumours

A

C Correct answer

C – Posterior mediastinum

Scenario 3

Thyroid mass

A

D Correct answer

D – Superior mediastinum

Scenario 4

Page 59: 51 100

Lymphoma

A Correct answer

A – Anterior mediastinum

Scenario 5

Bronchogenic cyst

A

B Correct answer

B – Middle mediastinum

THEME: Renal tract pathologies

A Adenocarcinoma of the kidneyB Adenoma of the renal cortexC Angioma of the renal arteryD AngiomyolipomaE NephroblastomaF NeuroblastomaG Papillary transitional cell tumour of the renal pelvisH Renal tuberculosisI Squamous cell carcinoma of the renal pelvisJ Transitional cell carcinoma of the bladderK Transitional cell tumour of the ureter

For each of the following statements, select the most likely cause for renal tract disease from the above list. Each option may be used once, more than once, or not at all.

 

Scenario 1

A 55-year-old smoker presents with a dragging discomfort in his left loin. He also gives a history of haematuria with occasional clot colic. On examination, a mass is felt over the left loin and he has a left-sided varicocele.

A Correct answer

Page 60: 51 100

A – Adenocarcinoma of the kidney

Adenocarcinoma of the kidney (hypernephroma; Grawitz’s tumour) affects more males than females (2 : 1) and is more prevalent in patients over 40 years of age. Risk factors include smoking, genetic factors, a high intake of fat, oil and milk, and exposure to toxins, such as lead, cadmium, asbestos and petroleum products. Clinical features include: a dragging discomfort in the loin and a triad of haematuria (with occasional clot colic), flank pain (in 35–40%) and palpable abdominal mass (in 25–45%). In men, a rapidly developing varicocele (most often on the left) is a characteristic sign. This is because the left testicular vein drains into the left renal vein, whereas the right testicular vein drains directly into the inferior vena cava. The patient may also manifest symptoms of hypertension, erythrocytosis and hypercalcaemia.

Scenario 2

A 20-month-old baby boy is brought to the paediatric surgical clinic by his mother who gives a history of failure to thrive, fever and occasional blood in the nappy. On examination, a soft mass that does not cross the midline is palpable on the right side of the abdomen.

A

E Correct answer

E – Nephroblastoma

Nephroblastoma (Wilms’ tumour) is a malignant mixed tumour seen in infancy. The tumours are usually solitary, soft, lobulated and are tan or grey in colour. The infant may present with pyrexia, haematuria (blood in the nappy), failure to thrive, and a non-tender abdominal (flank) mass. This mass does not cross the midline which distinguishes it from neuroblastoma (which usually crosses the midline), and is more nodular and irregular. Investigations include: blood count, biochemical profile, ultrasound scan (to confirm the mass and to also to view the other kidney), intravenous urogram (to give anatomical detail and an indication of renal function) and renogram. It is usually treated by total nephrectomy or partial nephrectomy (in children with bilateral disease) followed by radiotherapy.

Scenario 3

A 35-year-old man of Asian origin presents with evening rise of temperature, weight loss, increased urinary frequency and painful micturition. Urine investigation reveals a sterile pyuria.

A

Page 61: 51 100

H Correct answer

H – Renal tuberculosis

Renal tuberculosis commonly occurs in the 20–40-years age group; it is more common in males than females (2 : 1) and the right kidney is affected more than the left. The symptoms are an increase in the urinary frequency (both during the day and night), painful micturition, renal pain and haematuria. Constitutional symptoms are weight loss and a slight evening rise in temperature. Chemotherapy (pyrazinamide, isoniazid, rifampicin) forms the basis of management of genitourinary tuberculosis. The antituberculous drugs have high urinary excretion rates.

Scenario 4

A 67-year-old smoker presents with a 5-month history of painless haematuria, increased frequency of micturition and loss of weight. He worked in the dye industry before his retirement.

A

J Correct answer

J – Transitional cell carcinoma of the bladder

Transitional cell carcinoma of the bladder usually occurs over the age of 50 and is more common in men. The aetiology of this condition includes cigarette smoking (more than 20 cigarettes/day has 2–6 times risk of developing bladder cancer), working in the aniline dye and rubber industry (because of excretion of -naphthyl-amine in the urine), schistosomiasis infestation of the bladder and long-term catheterisation in paraplegic patients. Patients may present with painless haematuria, dysuria, frequency and urgency of micturition. Investigations include urine microscopy and culture (to rule out any infection) and cystoscopy. Endoscopic resection of the mass followed by a 4–6 week course of radiotherapy to the bladder and the pelvic side walls is useful in treating most tumours. Combination regimens of cisplatin, methotrexate and vinblastine (and adriamycin in some cases) are useful in the treatment of metastatic disease.

THEME: Surgical incisionsA Gable B Gridiron (or skin crease modification)C Kocher D Lanz E Lateral thoracotomy F Left paramedian G Long midline H Lower midlineI Median sternotomy

Page 62: 51 100

J Right paramedianK Rutherford–MorrisonL ThoracoabdominalM Transverse ‘unilateral’ PfannenstielN Upper midline

The following are descriptions of patients requiring surgery. Please select the most appropriate incision for the required operation from the list. The items may be used once, more than once, or not at all.

 

Scenario 1

A 72-year-old woman presents to The Emergency Department with a 1-day history of a painful lump in the left groin and vomiting. On examination she has a hard, tender, irreducible mass just below the inguinal ligament.

A

M Correct answer

M – Transverse ‘unilateral’ Pfannenstiel

This patient has a strangulated femoral hernia that requires urgent surgical repair. The McEvedy (high) approach was classically based on a vertical incision made over the femoral canal and continued upwards above the inguinal ligment; however this frequently resulted in an unsightly scar. This has now been replaced with a transverse ‘unilateral’ Pfannenstiel incision, which can be extended to form a complete Pfannenstiel incision if a formal laparotomy is required. Alternative approaches include the Lockwood (low), Lothiessen (transinguinal) and laparoscopic approaches. NB Hans Hermann Johannes Pfannenstiel, German gynaecologist (1862–1909).

Scenario 2

A 21-year-old man presents with a 3-day history of worsening lower right abdominal pain associated with anorexia. On examination, he is pyrexial and tachycardic. He has rebound tenderness in the right iliac fossa.

A

B Correct answer

B – Gridiron

Page 63: 51 100

The most likely diagnosis in this patient is appendicitis. Appendicectomy is an emergency procedure and the safest incision is a skin-crease modification of the oblique gridiron centred over McBurney’s point. It is the Editor’s view that attempts at removing the appendix via a Lanz incision can lead to significant peri-operative swelling, especially with a high-lying caecum in an adult, and place the patient at risk. If it is the desperate concern of the surgeon to avoid a visible scar then consideration should be given to performing the case laparoscopically.

Scenario 3

A 38-year-old woman has been taking regular diclofenac for relief of chronic backache. She is referred to The Emergency Department with sudden onset of upper abdominal pain. On examination she is pyrexial and tachycardic, abdominal examination reveals severe epigastric tenderness. There is a pneumoperitoneum evident on an erect chest X-ray.

A

N Correct answer

N – Upper midline

The most likely diagnosis in this patient is a perforated peptic ulcer. In those patients suitable for surgery, access is most commonly by upper midline incision. The historical alternative of a right paramedian incision is now rarely used, providing little additional exposure and potentially rendering part of the abdominal wall anaesthetic and ischaemic with poor wound healing and increased risk of hernias.

THEME: Local anaesthetic repair of an inguinal herniaA Femoral nerveB Genital branch of the genitofemoral nerveC Femoral branch of the genitofemoral nerveD Obturator nerveE Ilio-inguinal nerve

Which of the above nerves would result in:

The femoral nerve (L2–4) passes through the substance of the psoas, and then under the inguinal ligament a finger’s breadth lateral to the femoral artery. Its branches are muscular to the anterior compartment of the thigh (quadriceps, sartorius, pectineus), cutaneous (skin of the medial side of leg, ankle, foot to the great toe through the saphenous nerve), and articular to the hip and knee joint. The ilio-inguinal nerve is the collateral branch of the ilio-hypogastric nerve, with a terminal distribution to the skin of the root of the penis and anterior third of the scrotum, and a

Page 64: 51 100

small area below the medial end of the inguinal ligament. The genitofemoral nerve is derived from the first and second lumbar nerves but only fibres from L1 pass into the femoral branch. It supplies skin over most of the femoral triangle. The genital branch enters the spermatic cord and does not reach the skin.

Scenario 1

Numbness of the scrotum and root of penis?

A

E Correct answer

Scenario 2

Inability to extend the knee?

A Correct answer

THEME: Radial nerve injuryA Compression at the elbowB Fracture of the mid-humerusC Axillary compressionD Laceration at the wrist

Describe the level of injury. Pick the most appropriate option from the above list. Each option may be used once only, more than once or not at all.

In low radial nerve lesions, ie those due to fractures or dislocations at the elbow, the posterior interosseus nerve may be injured and the patient is unable to perform finger extension with weakness of thumb abduction and extension.In high lesions with fractures of the humerus or due to prolonged tourniquet pressure, there is weakness of the radial extensors of the wrist and numbness over the anatomical snuffbox. In very high lesions, the radial nerve may be compressed in the axilla, eg crutch palsy.

Scenario 1

Page 65: 51 100

A 25-year-old man presenting with weakness of the wrist and hand with paralysis of the triceps muscle and an absent triceps reflex.

A

C Correct answer

Axillary compression

Scenario 2

A 25-year-old man presenting with a wrist drop with inability to extend the metacarpophalangeal joints together with paraesthesia of the skin over the anatomical snuffbox.

A

B Correct answer

Fracture of the mid-humerus

Scenario 3

A 25-year-old man presenting with failure of extension of the metacarpophalangeal joints with weakness of thumb abduction and interphalangeal extension.

A Correct answer

Compression at the elbow

THEME: Diseases of the anusA Anal carcinomaB Anal intra-epithelial neoplasiaC Anal fissureD Anal fistulaE Condylomata acuminataF Fibroepithelial anal polypG HaemorrhoidsH Peri-anal abscessI Peri-anal haematomaJ Pilonidal abscessK Proctalgia fugax

Page 66: 51 100

L Skin tagsM Solitary rectal ulcer syndrome

The following are descriptions of local anorectal disorders. Please select the most appropriate diagnosis from the list. The items may be used once, more than once, or not at all.

 

Scenario 1

A 24-year-old man presents with a 3-month history of pain and passage of fresh blood on defaecation. Examination reveals a small skin tag at the anal verge; attempted proctoscopy has to be abandoned because of patient discomfort.

A

C Correct answer

C – Anal fissure

This is the typical presentation of this condition.

Scenario 2

A condition associated with chronic infection with human papillomavirus (especially serotypes 16 and 18).

A Correct answer

A - Anal Carcinoma, B - Anal intra-epithelial neoplasia, or E - Condylomata acuminata

Infection with human papillomavirus can lead to anal warts (molluscum contagiosum) and dysplastic changes within the anal epithelium (mild to severe: termed anal epithelial neoplasia). These may progress to anal carcinoma. So patients with warts and those with other sexually transmitted diseases affecting the anus should have biopsies and possibly thence surveillance if required.

Scenario 3

Page 67: 51 100

A 31-year-old man presents with a 1-year history of severe anal pain lasting for 2 to 3 minutes each night. Per rectum and proctosigmoidoscopic examinations are unremarkable.

A

K Correct answer

K – Proctalgia fugax

This is defined as episodic, intense anal pain of short duration (usually at night) in which all other disorders have been excluded. Proctalgia fugax occurs in up to 18% of the US population, being more common in men, and those < 40 years old. It is thought to be secondary to sensory dysfunction, with possible hypersensitivity of the internal anal sphincter and rectal musculature, precipitated by psychological stress. Treatment can be problematic with many systemic (eg antidepressants) and local (eg glyceryl trinitrate) remedies tried.

THEME: Common disorders of the hand

A Carpal tunnel syndrome B Dupuytren’s disease C Extensor tendon injury D Flexor tendon injury E Gamekeeper’s thumb F Ganglion G Heberden’s node H Mallet fingerI ParonychiaJ Phalangeal endochondromaK Pulp space infectionL Pyogenic granulomaM Rheumatoid arthritisN Trigger finger

The following patients present with disorders of the hand. From the list above, select the most likely diagnosis. The items may be used once, more than once, or not at all.

 

Scenario 1

A 44-year-old man attends the clinic and reports that his index finger often gets stuck and ‘clicks’ when he straightens it. He denies any history of trauma. He has no relevant past medical history. On examination, the finger, and rest of the hand,

Page 68: 51 100

appears normal. There is no contracture of the skin or subcutaneous tissues. Initially, the index finger is fixed in flexion, but it suddenly extends fully during active movement. There is no associated pain.

A

N Correct answer

N – Trigger finger

This condition is caused by thickening of the flexor tendon, paratenon, or a narrowing of the flexor sheath. Consequently, the affected finger becomes locked in full flexion and will only extend after excessive voluntary effort, or assistance from the other hand. When extension begins it does so suddenly, and with a click, hence the name of the condition. The condition is usually painless. Steroid infiltration may be effective in mild cases, although surgical release of the proximal portion of the A1 pulley may be necessary.

Scenario 2

A 64-year-old woman attends The Emergency Department with a fracture of the distal phalanx of her left middle finger. She informs you that this finger has become increasingly painful over the last few months, and that she has been aware of a ‘bony swelling’ affecting this finger. She indicates that this was most marked on the volar aspect of the distal phalanx, close to the distal interphalangeal joint.

A

J Correct answer

J – Phalangeal endochondroma

This benign tumour is composed of mature, hyaline cartilage, and presents as a slow-growing mass within a phalanx. Pain, swelling or deformity of the affected finger may be evident. The presentation may be acute, with a ‘pathological’ fracture through the weakened cortex, as in the case described. There is a characteristic appearance on radiography. The usual opacity of the bony phalanx is lost, and the cavity of the mass appears radiolucent with stippled calcification. The cortex of the bone may be thinned as the internal mass expands. Treatment involves curettage followed by cancellous bone grafting.

Scenario 3

A 28-year-old manual labourer attends The Emergency Department with a painful left index finger. He sustained a minor abrasion to the palmar aspect of his left index

Page 69: 51 100

finger at work 6 days ago. On examination, there is erythema and swelling affecting the distal aspect of the left index finger on the palmar surface.

A

K Correct answer

K – Pulp space infection

Pulp space infections usually arise from minor penetrating injuries. Pressure in the infected compartment causes marked pain. Infection may spread into adjacent compartments because of infarction of surrounding tissues secondary to rapidly increasing pressure. Occasionally, this may lead to rupture through the overlying skin, or into the distal phalanx. Treatment should involve early incision and drainage, to avoid permanent loss of pulp tissue, and subsequent reduction of cushioning of the distal phalanx.

THEME: Types of scientific evidenceA Case–control seriesB Case seriesC Cohort studyD Controlled clinical trialE Cross-sectional surveyF Experimental research paperG GuidelinesH Meta-analysisI Non-systematic reviewJ Randomised controlled trialK Randomised double-blind controlled trialL Systematic reviewM Uncontrolled trial

The following are descriptions of types of scientific evidence. Please select the most appropriate descriptive term from the list above. The items may be used once, more than once, or not at all.

(Resources for this subject may be obtained from the BMJ book by Trisha Greenhalgh entitled How to read a paper.)

Scenario 1

A statistical synthesis of the numerical results of several trials which all addressed the same question.

A

H Correct answer

Page 70: 51 100

When reviewing the impact of new therapies, evidence can come from

several studies of modest size and with slightly differing conclusions. One

solution might be to carry out a definitive randomised controlled trial but

this might require considerable time, effort and expense. An alternative is

to combine data from several modest studies into a meta-analysis. By

combining studies in a coherent (statistically robust) way, conclusions can be reached on a larger pool of subjects.

Scenario 2

A study in which two or more groups of people are selected on the basis of differences in their exposure to a particular agent and are followed-up to observe any differences in outcome between the groups.

A

C Correct answer

This differs slightly from a randomised controlled trial in that it generally

takes two or more large cohorts of subjects (rather than a specific sample

size of patients) and follows them up long term for the effects of a certain

agent, eg an environmental factor on the basis of which they are selected.

An example is Sir Richard Doll’s work associating lung cancer with smoking by observing a cohort of 40,000 doctors in four cohorts according to number of cigarettes smoked over 10 years.

Scenario 3

A study in which medical histories of more than one patient with a particular condition are described to illustrate one interesting aspect of the condition or treatment.

A

B Correct answer

Page 71: 51 100

This is synonymous with a series of case reports which together illustrate

an interesting aspect of a condition or treatment. Although they are not

randomised, are rarely prospective, and represent a low relative weight in

the traditional hierarchy of evidence, they are easy for the less scientifically

minded to digest and can still convey very important information rapidly

before a definitive trial can be performed, eg McBride’s 1961 case series

of two infants with limb absence born to mothers taking Thalidomide first

alerted the world to this terrible drug complication.

(Resources for this subject may be obtained from the BMJ book by Trisha Greenhalgh entitled How to read a paper.)

THEME: Groin lumps A Ectopic testis B Femoral artery aneurysm C Femoral hernia D Hydrocoele of the spermatic cordE Inguinal hernia F Inguinal lymphadenopathyG LipomaH Pseudo-aneurysmI Psoas abscessJ Psoas bursaK Saphena varixL Sarcoma 

The following patients all present with a lump in the groin. For each scenario please select the most appropriate diagnosis from the above list. The items may be used once, more than once, or not at all.

 

Scenario 1

A 36-year-old Asian immigrant presents to The Emergency Department with a tender, fluctuant mass in his left femoral triangle. He gives a history of night sweats, weight loss and a painful left hip.

Page 72: 51 100

A

I Correct answer

I – Psoas abscess

Although this mass could be attributed to lymphadenopathy (probably secondary to his clear history of tuberculosis), the fluctuant nature and the presence of ipsilateral hip pain point more readily to symptoms of a psoas abscess. Psoas abscesses develop either from infection of unknown origin or as a consequence of infection spreading from an adjacent organ. The risk factors for primary psoas abscess are not known; however, trauma to the muscle may be an important factor in 18–20% of cases. Low socioeconomic class and poor nutrition have also been cited as possible predisposing factors. A major risk factor for secondary psoas abscess is gastrointestinal pathology (inflammatory bowel disease, appendicitis, diverticulitis, bowel cancer and Crohn’s disease), and the source is a gastrointestinal infection in 80% of individuals. Prior to modern anti-tuberculous therapy, psoas abscesses occurred in up to 20% of patients with spinal tuberculosis. Treatment is now usually (initially at least) by percutaneous drainage under ultrasound or computed tomography guidance.

Scenario 2

A 68-year-old man attends Casualty suffering from drowsiness and confusion. His wife reports a 12-h history of vomiting and abdominal pain. On examination he is clearly dehydrated, his abdomen is distended and he has high-pitched bowel sounds. More detailed assessment reveals a small painful swelling in his right groin crease.

A

C Correct answer

C – Femoral hernia

This gentleman has signs of small bowel obstruction secondary to a strangulated femoral hernia. A small complicated hernia present in the groin crease in the elderly with no prior history of a reducible lump is much more likely to be a femoral than an inguinal hernia (although inguinal hernias are approximately ten times more common in general).

Scenario 3

A 62-year-old claudicant returns to the ward from the vascular assessment laboratory, following an angiogram of his lower limbs. The nurse is concerned about a swelling in his left groin. On closer examination you note a firm mass with a transmissible pulse.

Page 73: 51 100

A

H Correct answer

H – Pseudo-aneurysm

The nature of this man’s investigation points to the diagnosis of a pseudoaneurysm. Failure to compress the site of arterial cannulation (a traumatic breach to the vessel wall) leads to extravasation of arterial blood. A haematoma then forms in the soft tissues around the artery, which produces a transmissible pulse. A true aneurysm is one that involves all the layers of the arterial wall (three layers) and is described as being expansile.

THEME: type of intervention

A Best medical treatment and exercise

B Angioplasty and stenting

C Aorto-bifemoral graft

D Axillo-bifemoral graft

E Amputation

For each of the patients below, choose the most appropriate management from the list above. Each option may be selected once, more than once or not at all.

Patients with claudication do not require radiological/surgical intervention unless it interferes with their life style and they are willing to accept the small but significant risks associated with the procedures involved. Iliac angioplasty/stenting does achieve very good results with low complication rates in those patients with short occlusions. Patients with critical limb ischaemia require revascularisation to prevent them from loosing their leg in the near future. What they are offered depends on the length of the occlusion (surgery vs angioplasty) and on their level of fitness (aortic vs extra-anatomical bypass). Patients who are unlikely to walk in the future may be better managed with a primary amputation.

Scenario 1

A 60-year-old patient; no significant past medical history; bilateral rest pain; complete occlusion of both common and external iliac arteries

Page 74: 51 100

A

C Correct answer

Scenario 2

An 82-year-old patient; bed-bound secondary to right-sided cerebro-vascular accident; extensive gangrene of right foot; complete occlusion of both common and external iliac arteries

A

E Correct answer

Scenario 3

A 79-year-old patient; history of coronary heart disease and impaired renal function; bilateral gangrenous toes; complete occlusion of both common and external iliac arteries

A

D Correct answer

Scenario 4

A 70-year-old patient; sedentary life style; claudicates at 400 yards; short occlusions of both common iliac arteries

A Correct answer

Scenario 5

A 79-year-old patient; enthusiastic golfer; claudicates at 100 yards; short occlusions of both common iliac arteries

A

B Correct answer

Page 75: 51 100

THEME: CONDITIONS OF THE PAROTID GLAND

A Mumps B Carcinoma C Acute parotitisD Sjögren's syndrome E Mikulicz's syndrome F Pleomorphic adenoma G Sialectasis

For each of the patients described below, select the single most likely pathological condition from the options listed above. Each option may be used once, more than once, or not at all.

 

Scenario 1

An 80-year-old emaciated homeless man is admitted with a tender, erythematous swelling in the left parotid region. He is pyrexial.

A

C Correct answer

Acute parotitis is more common in the elderly and debilitated and is associated with poor oral hygiene, dehydration, and obstruction of Stensen’s duct by stone or scar tissue. There is pain and swelling of the parotid and the patient is also systemically unwell.

Scenario 2

An 18-year-old man is admitted with bilateral testicular swelling, upper abdominal pain, fever, malaise, and bilateral parotid swelling.

A Correct answer

Mumps is a viral parotitis typically presenting with bilateral painful glands and excessive oedema in young patients. In men it can also affect the testes.

Scenario 3

Page 76: 51 100

A 34-year-old woman presents with post-prandial discomfort and swelling of the left parotid salivary gland. She also has been experiencing dry eyes and joint stiffness.

A

D Correct answer

Sjögren’s syndrome is an autoimmune disorder affecting the salivary glands, and eventually both the parotid and submandibular glands. It is also characterised by dry mouth and dry eyes, as well as generalised arthritis.

THEME: LOWER LIMB TRAUMA

A Anterior compartment syndromeB Posterior compartment syndromeC Venous bleedD Traumatic nerve damageE Deep venous thrombosisF Lateral compartment syndrome

For each of the patients described below, select the single most likely diagnosis from the options listed above. Each option may be used once, more than once, or not at all.

 

Scenario 1

A motorcyclist rides into a lamppost and is unable to bear weight on his left leg. Lachman’s test is positive, there is no bony injury on X-ray, plantar flexing of the ankle causes considerable pain and the foot is cold and numb on palpation.

A

B Correct answer

This patient is likely to have posterior compartment syndrome, as the muscles of the posterior compartment are involved in plantar flexion.

Scenario 2

A patient suffers a displaced tibial fracture following a road traffic accident, is admitted and intramedullary nailing is done. You are on call and are asked to see him after he complains of severe pain in the leg that was operated on. He has severe pain

Page 77: 51 100

on passive flexion of the toes, and also complains of paraesthesia in the first web space.

A Correct answer

This patient is likely to have a compartment syndrome. The deep peroneal nerve which supplies the first web space runs in the extensor or anterior compartment of the leg.

THEME: Post-operative oliguria A Acute renal failureB Cardiogenic shockC Covert haemorrhageD Increased ADH secretionE Obstructed urinary catheterF Overt haemorrhageG SepsisH Severe diarrhoeaI Severe vomitingJ Third space lossK Urinary retention 

The above are all causes of post-operative oliguria. For the following scenarios please pick the most appropriate answer from the list. Each item may be used once, more than once, or not at all.

 

Scenario 1

A 64-year-old man has undergone surgery for repair of his 6.5-cm abdominal aortic aneurysm. He is recovering on the High Dependency Unit when the nurses call you to inform you that he has passed only 5–10 ml/hr of urine for the past 6 h, despite catheter flushing. On examination you note that his abdomen is not distended, his blood pressure is 130/75 mmHg; his pulse is 95/min and his central venous pressure is 12 cmH2O. You are awaiting blood results. The operative note reveals that the origin of the aneurysm was at vertebral level L2.

A Correct answer

A – Acute renal failure (ARF)

This man has suffered ARF as a complication of his aortic aneurysm surgery. The origin of his aneurysm in the scenario is at L2 – around the origin of the renal arteries (L1–L2). Inadvertent damage during surgery or occult embolism, after fresh or

Page 78: 51 100

organised thrombus is dislodged from the aneurysmal sac, can lead to acute renal failure.

Scenario 2

A 60-year-old diabetic man is recovering from a left femoropopliteal bypass. The nurses call you to let you know that he has failed to pass more than 5 ml/h of urine for the past 3 h. They have been giving him Gaviscon for ‘indigestion’ which has not settled. On examination you note that he is pale and sweaty. His blood pressure is 95/60 mmHg and his pulse is 98/min. The bandages covering his operative wound are minimally blood-stained and his thigh, although slightly swollen, is soft. Two fluid challenges each of 250 ml of Gelofusin fail to improve his blood pressure.

A

B Correct answer

B – Cardiogenic shock

This gentleman is diabetic and an arteriopath, as indicated by his admission for vascular bypass surgery. His poor urine output, compromised blood pressure (with no clear evidence of overt or covert haemorrhage) and the vague history of ‘indigestion’-type pain that has not settled with antacids, suggest a coronary cause for his hypotension and oliguria. This is confirmed by a failure of fluid bolus replacement to sustain a rise in blood pressure. This man needs thorough cardiovascular assessment (including electrocardiogram, chest X-ray, general bloods, cardiac enzymes and probably central venous pressure measurement). If he has sustained a myocardial infarct clearly he will be unable to undergo thrombolysis: such recent vascular surgery contra-indicates this. Instead treatment should be supportive (oxygen, analgesia), and treatment with inotropes should be initiated if his hypotension fails to improve on the Coronary Care Unit.

THEME: Common fracture eponyms A Barton’s fracture B Bennett’s fracture C Colles’ fracture D Galeazzi’s fracture dislocation E Garden II fracture F Garden III fracture G Garden IV fracture H Hill–Sachs fracture I Lisfranc fracture dislocationJ Monteggia’s fracture dislocationK Rolando’s fractureL Smith’s fractureM Weber A fracture

Page 79: 51 100

N Weber B fractureO Weber C fracture 

The following are descriptions of fractures. Please select the most appropriate fracture eponym from the above list. Each item may be used once, more than once, or not at all. These are all commonly used in current clinical practice (and so remain important).

 

Scenario 1

A comminuted, intra-articular fracture to the base of the first metacarpal.

A

K Correct answer

K – Rolando’s fracture

This description could be one of a Bennett’s fracture, but the comminuted nature distinguishes it as a Rolando’s fracture. There are usually three fragments forming either a Y-shape or a T-shape on radiograph.

Scenario 2

A distal fibular fracture at the level of the syndesmosis, with or without a malleolar fracture.

A

N Correct answer

N – Weber B fracture

The Weber (or Danis–Weber) classification describes the severity of tibio- fibular ligament injury by the level of fibular fracture. This is a Weber B fracture as it is sited at the level of the syndesmosis. A Weber A fracture is infra-syndesmotic while a Weber C fracture occurs above the level of the syndesmosis. The Lauge–Hansen classification is an alternative method of describing fractures of the distal tibia and fibular. It takes into account foot position and direction of deforming forces, and is preferred by senior orthopaedic surgeons. For your purposes the Weber system is sufficient as the Lauge–Hansen is complex and not all fractures fit the classical pattern.

Page 80: 51 100

Scenario 3

A complete fracture through the femoral neck, with rotation of the femoral head within the acetabulum, demonstrating minimal displacement.

A

F Correct answer

F – Garden III fracture

The Garden classification is used to describe intra-capsular fractures of the neck of the femur. It is important to distinguish these from extra-capsular fractures, as there is a bearing on blood supply, and ultimately, treatment. The capsule contributes the majority of the blood supply to the head of the femur, via the medial and lateral circumflex arteries from the profunda femoris. A compromise in the blood supply can result in avascular necrosis. The Garden system consists of four grades (I–IV) as follows:

Garden I – incomplete or impacted fracture Garden II – non-displaced fracture through both cortices Garden III – minimally displaced fracture with rotation of the femoral head in

the acetabulum Garden IV – completely displaced fracture of the head of femur (no continuity

between the proximal and distal fragments).

Scenario 4

An intra-articular fracture of the volar or dorsal margin of the distal radius. The fracture extends obliquely to the radio-carpal joint with a striking dislocation of the carpus.

A Correct answer

A – Barton’s fracture

This injury results from a fall onto an outstretched hand. The Barton fracture can be sub-divided into volar and dorsal types. It can be distinguished radiographically from Colles’ or Smith’s fractures by the presence of a dislocation or subluxation. This involves the rim of the distal radius, which can be dorsally or volarly displaced with the hand and carpus. The majority of these fractures require surgical reduction and fixation.

THEME: Micro-organisms

Page 81: 51 100

A Clostridium difficileB Clostridium perfringensC Escherichia coliD Haemophilus influenzaeE Staphylococcus aureusF Streptococcus pneumoniaeG None of the above

From the above list, choose the most common causative organism for the following infections. Each may be used once, more than once, or not at all.

 

Scenario 1

Ludwig’s angina

A

F Correct answer

F – Streptococcus pneumoniae

Ludwig’s angina is bilateral infection of the submandibular, sublingual and submental spaces – usually arising from dental sepsis. Streptococcal infection spreads in deep cervical and pharyngeal fascial planes. It may cause airway obstruction.

Scenario 2

Vincent’s angina

A

G Correct answer

G – None of the above

Vincent’s angina is due to the symbiotic action of fusiform bacteria and the spirochaete Borrelia vincentii. It is a pharyngeal infection with ulcerative gingivitis.

Scenario 3

Page 82: 51 100

Post-operative diarrhoea

A Correct answer

A – Clostridium difficile

After broad-spectrum antibiotic therapy, infection with Clostridium difficile may lead to pseudomembranous colitis.

THEME: Oesophageal diseases

A Gastro-oesophageal reflux disease (GORD)B Post-cricoid webC Hiatus herniaD Oesophageal carcinomaE AchalasiaF Barrett's oesophagusG Oesophageal stricture

For each of the following statements, select the most likely answer from the above list. Each option may be used once, more than once, or not at all.

Achalasia is due to degeneration of the myenteric plexus causing dysphagia and often aspiration. A post-cricoid web may predispose to carcinoma (usually squamous), as does achalasia and Barrett’s oesophagus (adenocarcinoma). Hiatus hernia may have normal lower oesophageal sphincter pressures, unless associated with gastro-oesophageal reflux disease (GORD).

Scenario 1

Metaplastic change of squamous to columnar epithelium

A

F Correct answer

Scenario 2

Shows the characteristic mega-oesophagus on barium swallow

Page 83: 51 100

A

E Correct answer

Scenario 3

Associated with iron deficiency anaemia

A

B Correct answer

Scenario 4

Characterised by a weak lower oesophageal sphincter

A Correct answer

Scenario 5

Manometry reveals high lower oesophageal sphincter pressure with failure to relax during swallowing

A

E Correct answer

THEME: Complications of cadaveric organ transplantation

A Acute rejection B Arterial thrombosis C Azathioprine side-effects D Chronic rejection E Cyclosporin side-effects F Cytomegalovirus infection G Graft-versus-host diseaseH Hyperacute rejectionI MalignancyJ Primary graft non-functionK Steroid side-effectsL Venous thrombosis

Page 84: 51 100

The following patients have all previously undergone cadaveric organ transplantation. From the above list, select the most likely complication. The items may be used once, more than once, or not at all.

 

Scenario 1

A 25-year-old woman with cystic fibrosis underwent a heart and lung transplant 4 months ago, and is currently receiving triple immunotherapy. She attends the follow-up clinic for a routine check-up. She reports sore gums and excessive facial hair. On examination she is hypertensive. Urea & electrolyte results are as follows: Na+ 139 mmol/litre, K+ 5.7 mmol/litre, urea 9.9 mmol/litre, creatinine 140 µmol/litre.

A

E Correct answer

E – Cyclosporin side-effects

Cyclosporin A is an example of a calcineurin inhibitor. Cyclosporin-based triple immunosuppression with corticosteroids and azathioprine remains the most popular regimen in the UK. It is used prophylactically and therapeutically to address rejection following organ transplantation. Sideeffects of cyclosporin include nephrotoxicity, hypertension, hirsutism, tremor, gingival hyperplasia and hepatotoxicity. Long-term use increases the risk of development of malignancy (5% of patients), most commonly basal or squamous cell carcinomas.

Scenario 2

A 58-year-old man had a liver transplant 7 weeks ago. He attends The Emergency Department with a 24-h history of malaise, fever and myalgia and respiratory distress going upstairs. He is currently receiving triple immunosuppression. On examination, he is unwell and dyspnoeic at rest. He has a pulse rate of 105/min, blood pressure is 95/60 mmHg, temperature is 38.3°C and respiratory rate is 28 breaths/min. Oxygen saturation is 90% on air.

A

F Correct answer

F – Cytomegalovirus infection

Page 85: 51 100

In addition to the development of malignancy, immunosuppression increases the risk of infection. Cytomegalovirus (CMV) is a member of the herpes group of viruses. Primary infection in a seronegative individual who receives a graft from a seropositive donor typically occurs 6 weeks posttransplantation, and results in the most severe disease. The main symptoms of CMV infection are usually non-specific and include fever, night sweats, fatigue and myalgia. Retinitis is pathognomonic, but rarely seen in the transplant population. Respiratory distress, noticed at first during exercise, may give a clue to early CMV pneumonitis. Patients may also present with CMV encephalitis or gastrointestinal infection, characterised by dysphagia, diarrhoea, nausea and abdominal pain. Reactivation of latent CMV infection may also occur in immunosuppressed patients, although the infection is usually less severe.

Scenario 3

A 41-year-old woman underwent renal transplantation 18 months ago. She attends follow-up clinic and is currently asymptomatic. On examination her blood pressure is 150/110 and urinalysis reveals 3+ protein. Urine culture is negative. A renal ultrasound scan reveals a normal collecting system. Renal biopsy demonstrates intrarenal arteriosclerosis with associated glomerular atrophy and interstitial fibrosis.

A

D Correct answer

D – Chronic rejection

Chronic rejection is characterised clinically by a progressive deterioration in graft function occurring months to years after transplantation and is associated with typical histological changes of graft atherosclerosis and fibrosis. By contrast, acute rejection occurs within the first 3 months after transplantation, and hyperacute rejection occurs within hours. For the diagnosis of chronic rejection to be made, other causes of graft dysfunction must be excluded (eg infection, calcineurin antagonist toxicity, etc), and a transplant graft biopsy is required to confirm the diagnosis histologically. For renal transplantation, graft dysfunction is manifested by a rise in serum creatinine as a result of progressive decline in the glomerular filtration rate. There is associated proteinuria and worsening hypertension, with the diastolic component classically rising in advance of the systolic component

THEME: Peripheral nerve injuryA Femoral nerveB Common peroneal nerveC Deep peroneal nerveD Superficial peroneal nerveE Sural nerveF Tibial nerveG Saphenous nerve

Page 86: 51 100

Pick the most appropriate option from the above list. Each option may be used once only, more than once or not at all.

 

Scenario 1

A 28-year-old man sustains a varus injury to his left knee while skiing. He notes loss of sensation over front and anterior half of the leg and dorsum of the foot.

A

B Correct answer

Common peroneal nerve

Deep peroneal nerve injury may give rise to an anterior compartment syndrome and loss of sensation in the first web space between the first and second toes. The common peroneal nerve is often damaged at the level of the fibular neck by severe traction when the knee is forced into varus (eg lateral ligament injuries and fractures around the knee) or from pressure from a splint or plaster cast. The patient has foot drop and loss of sensation over the front and outer half of the leg and dorsum of the foot.

Scenario 2

A 32-year-old motorcyclist is involved in a road traffic accident and sustains a severe laceration 6 cm above the ankle on the lateral aspect of his leg. He is unable to evert his foot and has noted some numbness over the dorsum of foot and medial four toes.

A

D Correct answer

Superficial peroneal nerve

The superficial peroneal nerve innervates the peroneal muscles and emerges through the deep fascia 5-10 cm above the ankle to supply the skin over the dorsum of the foot and medial four toes.